Vous êtes sur la page 1sur 43

Test No.

IAS Prelims - 2020 16


PTS2020/PPP-16/112019/26

GENERAL STUDIES

RE
CURRENT AFFAIRS
(SEPTEMBER + OCTOBER 2019)

O
Answer Key

Q. 1 (a) Q. 21 (b) Q. 41 (a) Q. 61 (a) Q. 81 (d)


SC
Q. 2 (b) Q. 22 (b) Q. 42 (c) Q. 62 (c) Q. 82 (a)
Q. 3 (b) Q. 23 (b) Q. 43 (a) Q. 63 (a) Q. 83 (a)
Q. 4 (c) Q. 24 (b) Q. 44 (a) Q. 64 (b) Q. 84 (c)
Q. 5 (b) Q. 25 (d) Q. 45 (c) Q. 65 (c) Q. 85 (a)
Q. 6 (b) Q. 26 (c) Q. 46 (c) Q. 66 (d) Q. 86 (a)
GS

Q. 7 (b) Q. 27 (a) Q. 47 (c) Q. 67 (b) Q. 87 (d)


Q. 8 (c) Q. 28 (b) Q. 48 (c) Q. 68 (d) Q. 88 (c)
Q. 9 (c) Q. 29 (d) Q. 49 (b) Q. 69 (c) Q. 89 (d)
Q. 10 (d) Q. 30 (a) Q. 50 (d) Q. 70 (a) Q. 90 (b)
Q. 11 (c) Q. 31 (d) Q. 51 (b) Q. 71 (a) Q. 91 (c)
Q. 12 (b) Q. 32 (b) Q. 52 (d) Q. 72 (c) Q. 92 (d)
Q. 13 (b) Q. 33 (b) Q. 53 (a) Q. 73 (a) Q. 93 (d)
Q. 14 (d) Q. 34 (c) Q. 54 (a) Q. 74 (d) Q. 94 (c)
Q. 15 (c) Q. 35 (c) Q. 55 (c) Q. 75 (c) Q. 95 (b)
Q. 16 (b) Q. 36 (d) Q. 56 (d) Q. 76 (a) Q. 96 (a)
Q. 17 (c) Q. 37 (a) Q. 57 (b) Q. 77 (c) Q. 97 (b)
Q. 18 (b) Q. 38 (d) Q. 58 (a) Q. 78 (c) Q. 98 (c)
Q. 19 (c) Q. 39 (b) Q. 59 (c) Q. 79 (b) Q. 99 (a)
Q. 20 (b) Q. 40 (d) Q. 60 (d) Q. 80 (a) Q. 100 (d)

1 PTS2020/PPP-16/112019/26
PTS2020/PPP-16/112019/26
1. Correct Option: (a)  Its prime objective is enforcement of two
key Acts of Government of India namely:
Explanation:
 Foreign Exchange Management Act
 Option (a) is correct: Sabka Vishwas 1999 (FEMA) and
Scheme is a Legacy Dispute Resolution
 Prevention of Money Laundering Act
scheme is offered to those taxpayers who
2002 (PMLA)
wish to disclose any previously undisclosed
tax liability without any penalty or  To check money laundering by curb black
prosecution. money and hawala trade cases.

Supplementary notes:  It is composed of officers from the Indian


Revenue Service, Indian Police Service and
Sabka Vishwas Scheme the Indian Administrative Service.
 The scheme targets those taxpayers who  It was established in 1956 as ‘Enforcement
want to close their pending disputes related Unit’ in Department of Economic Affairs.
to Service Tax and Excise Tax (now
 It is headquartered in New Delhi.
subsumed under Goods and Services Tax).
 The two main components of the Scheme
are dispute resolution and amnesty. 3. Correct Option: (b)

RE
 The dispute resolution component Explanation:
is aimed at liquidating the legacy cases  Statement 1 is incorrect: National Sports
of Central Excise and Service Tax that Day is celebrated on the birth anniversary
are subsumed in GST and are pending of hockey legend Major Dhyan Chand.
in litigation at various forums.
Supplementary notes:
 The amnesty component of the Scheme
Fit India Movement
CO
offers an opportunity to the taxpayers
to pay the outstanding tax and be free  Prime Minister has launched nation-
of any other consequence under the wide Fit India Movement on the
law. The most attractive aspect of the occasion of National Sports Day.
Scheme is that it provides substantial
relief in the tax dues for all categories of  Fit India Movement aims to motivate every
cases as well as full waiver of interest, Indian to incorporate simple, easy ways of
fine, penalty, In all these cases, there staying fit in their everyday life.
would be no other liability of interest, National Sports Day
S

fine or penalty. There is also a complete


amnesty from prosecution.  It is celebrated on 29 August, on
the birth anniversary of hockey
 The Scheme is especially tailored to free legend Major Dhyan Chand.
the large number of small taxpayers
GS

of their pending disputes with the tax  On this Day, President confers the National
Sports Awards, National Adventure
administration.
Awards, Arjuna Award, KhelRatna,
Dronacharya Award and Dhyanchand
2. Correct Option: (b) Award to recognise the exceptional
achievements of Indian sportspersons.
Explanation:
 Statement 1 and 4 are incorrect: 4. Correct Option: (c)
Enforcement Directorate is responsible
for the administration of Prevention Explanation:
of Money Laundering Act, 2002 and  Both statements are correct
Foreign Exchange Management Act,
1999 only. Supplementary notes:
Supplementary notes: RBI Economic Capital Framework

Enforcement Directorate  Recently, the Reserve Bank of India


(RBI) accepted the recommendations
 It is economic intelligence and law of Bimal Jalan panel; set up to review the
enforcement agency responsible for enforcing RBI’s Economic Capital Framework.
economic laws and fighting economic crime
in India.  Following these recommendations, RBI
decided to transfer Rs 1.76 lakh crore in
 It functions under aegis of Department of dividend and surplus reserves to the
Revenue, Union Ministry of Finance. government.

PTS2020/PPP-16/112019/26 2
 The objective of the economic capital  This framework would help the
framework is to build harmony industry reduce its carbon emissions,
between the central bank’s need for increase resource efficiency, tackle waste
autonomy and the Government’s and water management, and create
objectives of the development. positive social impact to achieve long-term
sustainability targets.
 The Bimal Jalan-led panel
recommended holistic risk capital
frameworks to assess the adequacy of RBI 6. Correct Option: (b)
reserves. Some recommendations of the
Explanation:
committee are:
 Statement 1 is incorrect: The 45th
 RBI to maintain the Contingency
WorldSkills Competition were held at the
Risk Buffer (CRB), which is the KAZAN EXPO International Exhibition
country’s fund to handle financial Centre in Kazan, Russia.
stability within the range of 5.5% to
6.5% of the RBI’s balance sheet. Supplementary notes:
 It recommended a review of the WorldSkills Competition
RBI’s Economic Capital Framework
 The 45th WorldSkills Competition was
(ECF) every five years. held at the KAZAN EXPO International

RE
 The RBI’s accounting year of July- Exhibition Centre in Kazan, Russia.
June can be brought in sync with  It is a flagship event of the WorldSkills
the fiscal year of April-March from the International, organised every two years.
financial year 2020-21.
 It is the biggest vocational
 The report has also removed the education and skills excellence event
interim payout structure in general O in the world that truly reflects global
circumstances. industry.
 All the recommendations of the  The aims of the competition
panel have been accepted by the include demonstrating the advantages of
RBI. learning a vocational skill, and encouraging
‘parity of esteem’ between vocational and
SC
academic qualifications.
5. Correct Option: (b)
Explanation: 7. Correct Option: (b)
 Statement 1 is incorrect: The SURE Explanation:
project is a commitment by India’s
apparel industry to set a sustainable  Statement 1 is incorrect: The Report was
released by the non-government organization
pathway for the Indian fashion
GS

World Vision India and research institute


industry.
IFMR LEAD.
Supplementary notes: Supplementary notes:
Project SURE Child well-being index
 The SURE project is a commitment  The India child well-being index is a
by India’s apparel industry to set a crucial report that can be mined both by
sustainable pathway for the Indian the Government and civil organisations to
fashion industry. achieve the goal of child well-being and we
 SURE stands for ‘Sustainable Resolution’ – will use this report effectively.
a firm commitment from the industry to  This report provides insights on health,
move towards fashion that contributes to a nutrition, education, and sanitation and
clean environment. child protection.
 The project has been launched by  The dimensions of the index include
the Union Textiles Ministry, along healthy individual development, positive
with Clothing Manufacturers Association of relationships and protective contexts.
India (CMAI); United Nations in India; and  Focusing on the three key dimensions,
IMG Reliance. 24 indicators were selected to develop the
 It will be the first holistic effort by the apparel computation of the child well-being index.
industry towards gradually introducing a  The index captures the performance of each
broader framework for establishing critical state and union territory on a composite
sustainability goals for the industry. child well-being score.

3 PTS2020/PPP-16/112019/26
 Kerala, Tamil Nadu, Himachal Pradesh and  The Government will examine the
Puducherry topped the charts in the child Expression of Interest and justification. If
well-being index. it is suitable for award, Govt. will call for
 Meghalaya, Jharkhand and Madhya competitive bids after obtaining necessary
Pradesh featured at the bottom. environmental and other clearances.

 Among the union territories, Puducherry  OALP was introduced vide a Cabinet
led the way and Dadra and Nagar Haveli decision of the Government as part of the
featured at the other end. new fiscal regime in exploration sector
called HELP or Hydrocarbon Exploration
 The report has called for states to look at
their respective scores on the dimensions of and Licensing Policy, so as to enable a
child well-being and to prepare for priority faster survey and coverage of the available
areas of intervention with specific plans of geographical area which has potential for
action. oil and gas discovery.
 The report also emphasised on triggering  What distinguishes OALP from New
policy level changes, seek better budgetary Exploration and Licensing Policy (NELP)
allocations and initiate discussions with all is that under OALP, oil and gas acreages
stakeholders which can help in enhancing will be available round the year instead of
the quality of life of all children in the cyclic bidding rounds as in NELP. Potential
country.

RE
investors need not have to wait for the
bidding rounds to claim acreages.
8. Correct Option: (c)  Setting up of National Data Repository is one
of the milestones achieved for Open Acreage
Explanation:
Licensing Policy. To make India a favorable
 Both statements are correct destination globally for Exploration of Crude
Oil and Natural Gas, the Government plans
Supplementary notes:
CO
to move to the OALP regime soon.
International Fund for Animal Welfare
 It is well acknowledged that there is a need
(IFAW)
for a faster vehicle of awarding blocks in
 The International Fund for Animal Welfare order to bring more area under exploration.
is a global non-profit helping animals and
people thrive together.  As India has vast unexplored sedimentary
basins, a strategy which facilitates a time
 Experts and everyday people, working bound full coverage has become a necessity.
across seas, oceans and in more than 40 Moreover, even the OALP pre-supposes
S

countries around the world. offering of data to the interested companies


 It rescue, rehabilitate and release animals, for them to submit their bids/ interest.
and restore and protect their natural Hence, availability of data is no longer an
habitats. option, but a pre-condition.
GS

 Partner with local communities,


governments, non-governmental 10. Correct Option: (d)
organizations and businesses.
Explanation:
9. Correct Option: (c)  All statements are correct
Explanation: Supplementary notes:
 Both statements are correct Food fortification in India
Supplementary notes:  Fortification is the addition of key
Open Acreage Licensing Policy (OALP) vitamins and minerals such as iron,
iodine, zinc, Vitamin A & D to staple foods
 Open Acreage Licensing Policy (OALP) gives such as rice, milk and salt to improve their
an option to a company looking for exploring nutritional content. These nutrients may or
hydrocarbons to select the exploration may not have been originally present in the
blocks on its own, without waiting for the
food before processing.
formal bid round from the Government.
 India’s National Nutritional strategy,
 Under Open Acreage Licensing Policy
(OALP), a bidder intending to explore 2017, had listed food fortification as one
hydrocarbons like oil and gas, coal bed of the interventions to address anaemia,
methane, gas hydrate etc., may apply to the vitamin A and iodine deficiencies apart
Government seeking exploration of any new from supplementation and dietary
block (not already covered by exploration). diversification.

PTS2020/PPP-16/112019/26 4
 Malnutrition is a prominent issue in India 11. Correct Option: (c)
— 38 per cent of children under five years
are stunted i.e. too short for their age, 36 Explanation:
per cent are underweight and 21 per cent  Statement 1 is incorrect: The scheme was
are wasted i.e. too thin for their height, first launched on a pilot basis in 2009-10
which is a sign of acute under-nutrition.
 Statement 3 is incorrect: It aimed
 59 per cent women and 53 per cent children to achieve integrated development of
are anaemic. selected villages with more than 50% SC
 The Food Safety and Standards population.
Authority of India (FSSAI) made Supplementary notes:
standards for fortification in the Food
Safety and Standards (Fortification of Pradhan Mantri Adarsh Gram Yojana
Foods) Regulations, 2018, for five staples —  The Union government has identified
wheat, rice, milk, oil and salt. 26,968 Dalit-majority villages for focused
 The standards are given for wheat and implementation of various government
rice fortification with iron, folic acid, schemes over the next five years.
and vitamin B12, the deficiency of which  All existing Central and state welfare
cause anaemia. Besides, other B vitamins

RE
schemes that are necessary to meet critical
are also added. gaps in socio-economic and infrastructure
 Standards are provided for oil and milk needs, and reduce disparities will be
fortification with vitamin A and implemented in Dalit villages.
vitamin D, the deficiency of which cause  These villages — where over 50 per cent of
night blindness and rickets respectively; the total population are Dalits and those
and salt fortification with iron along with
O with a sizeable overall population — have
iodine to prevent goitre. been zeroed in under the Pradhan Mantri
 The food companies who wish to add Adarsh Gram Yojana for “integrated
micro-nutrients to these staples sold in development”.
the packages will also have to follow the  While the scheme is mainly about ensuring
SC
standards set by FSSAI. If the product is focused implementation and convergence
fortified according to the standards, the of existing Central and state government
package will carry an F+ label. schemes, the ministry provides Rs 21 lakh
 The Union Ministries of Women and per village as gap funding.
Child Development, Human Resource  According to the 2011 Census, Dalits make
Development and Consumer Affairs, Food up 16.6 per cent of the country’s total
and Public Distribution have mandated the population, with states such as Punjab,
distribution of fortified wheat flour, Uttar Pradesh, Haryana, Himachal Pradesh,
GS

rice, oil and double fortified salt in their West Bengal, and the Union Territory of
schemes — Integrated Child Development Delhi recording more than 20 per cent Dalit
Services (ICDS) and Mid-Day Meal (MDM) population.
and Public Distribution System (PDS)
respectively.
12. Correct Option: (b)
Some of the important features of Food Safety
and Standards (Fortification of Foods)
Explanation:
Regulations, 2018 are as follows:  Statement 1 is incorrect: Pradhan Mantri
 It prescribes the standards of addition of micro- Mudra Yojana (PMMY) can be availed for
nutrients for the purpose of food fortification. non-farm sector income generating activities
only.
 The manufacturers of the fortified food are
required to provide a quality assurance Supplementary notes:
undertaking.
Pradhan Mantri MUDRA Yojana (PMMY)
 Packaging and labelling of the fortified food
 Pradhan Mantri Mudra Yojana (PMMY) is
must state the food fortificant added,
a flagship scheme of Government of India
logo and the tagline “Sampoorna Poshan
to “fund the unfunded” by bringing such
Swasth Jeevan”.
enterprises to the formal financial system
 It should be in compliance to the Food Safety and extending affordable credit to them.
and Standards (Packaging and Labelling)
 It enables a small borrower to borrow from
Regulations, 2011.
all Public Sector Banks such as PSU Banks,

5 PTS2020/PPP-16/112019/26
Regional Rural Banks and Cooperative  India has been allocated a site of 75,000
Banks, Private Sector Banks, Foreign sq km in the Central Indian Ocean Basin
Banks, Micro Finance Institutions (MFI) by the International Sea Bed Authority for
and Non-Banking Finance Companies exploration of polymetallic nodules from
(NBFC) for loans upto Rs 10 lakhs for non- seabed.
farm income generating activities.  The estimated resource of polymetallic
 The scheme was launched on 8th April, nodules is about 380 million tonnes,
2015 by the Hon’ble Prime Minister. containing 4.7 million tonnes of nickel, 4.29
million tonnes of copper and 0.55 million
Eligibility tonnes of cobalt and 92.59 million tonnes of
 Any Indian Citizen who has a business plan manganese.
for a non-farm sector income generating
activity such as manufacturing, processing, 14. Correct Option: (d)
trading or service sector and whose credit
need is less than Rs 10 lakh. Explanation:
 The citizen can approach eithera Bank,  All statements are correct
MFI, or NBFC for availing of Micro Units
Development & Refinance Agency Ltd. Supplementary notes:
(MUDRA) loans under Pradhan Mantri
Automatic Exchange of Information

RE
Mudra Yojana (PMMY).
(AEOI)
Types of loans provided  The new global standard on Automatic
Under the aegis of Pradhan Mantri MUDRA Exchange of Information (AEOI)
Yojana, MUDRA has already created the reduces the possibility for tax evasion.
following products / schemes.
 It provides for the exchange of non-resident
 Shishu : covering loans upto 50,000/- financial account information with the tax
authorities in the account holders’ country
CO
 Kishor : covering loans above 50,000/- and of residence.
upto 5 lakh
 Participating jurisdictions that
 Tarun : covering loans above 5 lakh and
implement AEOI send and receive pre-
upto 10 lakh
agreed information each year, without
having to send a specific request.
13. Correct Option: (b)  AEOI will enable the discovery of
formerly undetected tax evasion.
Explanation:
S

 It will enable governments to recover tax


 Option (b) is correct: The ‘Samudrayaan’
revenue lost to non-compliant taxpayers, and
is a pilot project of the Ministry of Earth
will further strengthen international efforts
Sciences’ for deep ocean mining for rare
minerals. to increase transparency, cooperation, and
GS

accountability among financial institutions


Supplementary notes: and tax administrations.
Samudrayaan  Additionally, AEOI will generate
secondary benefits by increasing voluntary
 The ‘Samudrayaan’ is a pilot project of the disclosures of concealed assets and by
Ministry of Earth Sciences for deep ocean encouraging taxpayers to report all relevant
mining for rare minerals. information.
 The project proposes to send a submersible
vehicle with three persons to a depth of about
6000 metres to carry out deep underwater
15. Correct Option: (c)
studies. Explanation:
 The indigenously developed vehicle is  Option (c) is correct: The Ministry of
capable of crawling on the sea bed at a depth Electronics and Information Technology
of six kilometre for 72 hours. (MeitY) and Google have signed a statement
 Similar to ISRO’s plan to carry persons of intent to roll-out ‘Build for Digital India’
on a space mission, NIOT has undertaken programme.
Samudrayaan. The ¹ 200 croreproject
Supplementary notes:
proposes to carry three persons in a
submersible vehicle to a depth of 6000 Build for Digital India Programme
metres under sea for various studies.
 The Ministry of Electronics and Information
 The success of the ‘’Samudrayaan’’ will help Technology (MeitY) and Google have signed
India join the league of developed nations in a statement of intent to roll-out ‘Build for
exploration of minerals from oceans. Digital India’ programme.

PTS2020/PPP-16/112019/26 6
About: Supplementary notes:
 The programme will give engineering POSHAN Abhiyaan
students a platform to develop technology-
based solutions to address key social  POSHAN Abhiyaan is a multi-
ministerial convergence mission with
problems.
the vision to ensure attainment of
 As part of the programme, engineering malnutrition free India by 2022.
students across the country will be invited
 The objective of POSHAN Abhiyaan to
to apply and join in a learning journey that reduce stunting in identified Districts
will help them transform their bright ideas of India with the highest malnutrition
into real-world solutions. burden by improving utilization of key
 Applicants will take part in online and Anganwadi Services and improving
offline learning opportunities on key the quality of Anganwadi Services
technologies such as machine learning, delivery.
cloud and android.  Its aim to ensure holistic development and
 These will be offered through Google’s adequate nutrition for pregnant women,
Developer Student Club network and other mothers and children.
Google Developer networks.  The Ministry of Women and Child
Development (MWCD) is implementing

RE
 Google will also offer mentorship sessions
POSHAN Abhiyaan in 315 Districts in
in product design, strategy and technology
first year, 235 Districts in second year
to the most promising products and
and remaining districts will be covered
prototypes.
in the third year.
 There are a number of schemes directly/
16. Correct Option: (b) indirectly affecting the nutritional status
of children (0-6 year’s age) and pregnant
Explanation:

head of the NWBL.
O
Option (b) is correct: PM is the ex-officio
women and lactating mothers. In spite of
these, level of malnutrition and related
problems in the country is high. There is
no dearth of schemes but lack of creating
Supplementary notes: synergy and linking the schemes with each
SC
other to achieve common goal. POSHAN
National Board for Wildlife Abhiyaan through robust convergence
 National board for wildlife recently gave its mechanism and other components would
approval to Gulmarg defence project. strive to create the synergy.
 The proposed project in J&K includes Feature:
installation of air defence and weaponry The POSHAN Abhiyaan, as an apex body,
along with helipad construction. “Only radar will monitor, supervise, fix targets and guide
system would be set up in the sanctuary the nutrition related interventions across the
GS

area and all other basic infrastructure will Ministries.


be outside the wildlife area”.
The proposal consists of
 The National Board for Wildlife (NBWL)
has given its nod for an air defence and  mapping of various Schemes contributing
weaponry project in Jammu and Kashmir towards addressing malnutrition
in which 1.18 hectares of Gulmarg Wildlife  introducing a very robust convergence
Sanctuary land will be used. The project mechanism
will also cover 12.35 hectares of forest land
outside Gulmarg Sanctuary.  ICT based Real Time Monitoring system
 incentivizing States/UTs for meeting the
 The proposal, recommended by the then
targets
State Chief Wildlife Warden with 17 riders/
conditions, was cleared at a July 18 meeting  incentivizing Anganwadi Workers (AWWs)
of the standing committee of NBWL. for using IT based tools
The meeting was chaired by Minister for
 eliminating registers used by AWWs
Environment, Forest and Climate.
 introducing measurement of height of
children at the AnganwadiCentres (AWCs)
17. Correct Option: (c)
 Social Audits
Explanation:  Setting-up Nutrition Resource Centres,
 Statement 2 is incorrect: The Ministry of involving masses through Jan Andolan for
Women and Child Development (MWCD) is their participation on nutrition through
implementing POSHAN Abhiyaan. various activities, among others.

7 PTS2020/PPP-16/112019/26
18. Correct Option: (b)  Under this scheme, IoEs will have freedom
to choose their own path to become world
Explanation: class institutions. Institutions shall be
 Statement 2 is incorrect: IoEs will be selected as IoEs by the Empowered Expert
Committee (EEC), constituted for this
free to fix and charge fees from foreign
purpose.
students without restriction
 The EEC shall conduct their appraisal
 Statement 3 is incorrect: Each Public
based on their 15 years strategic vision
Institution selected as IoE will be provided
plan and 5 years implementation plan as
financial assistance up to Rs. 1000 crore
well as any other measure of demonstrated
over a period of five years. commitment to the cause of developing an
Supplementary notes: Institution of Eminence.
 The EEC will make its recommendation
Institution of Eminence
after engagement with the Institutions to
 University Grants Commission (UGC) study their proposals, hear presentations
issued / notified enabling regulatory from them and after a thorough scrutiny,
architecture in the form of UGC (Declaration rank the Institutions for their suitability for
of Government Educational Institutions inclusion in this scheme.
as Institutions of Eminence) Guidelines,

RE
 Each Public Institution selected as IoE
2017 and UGC (Institutions of Eminence will be provided financial assistance
Deemed to be Universities) Regulations, up to Rs. 1000 crore over a period of
2017 to enable 10 public and 10 private five years.
Institutions to emerge as World Class
Teaching and Research Institutions. These
Institutions shall be called as ‘Institutions 19. Correct Option: (c)
of Eminence (IoEs)’.
CO
Explanation:
 The aim of the scheme is to bring higher
 Statement 4 is incorrect: MDR-TB is
educational institutions selected as
resistant to anti-TB drugs like isoniazid
IoEs in top 500 of world ranking in the and rifampicin
next 10 years and in top 100 eventually
overtime. Supplementary notes:
 The scheme has been launched with an Tuberculosis
objective to provide world class teaching
Rapid molecular tests: This tests is used
S


and research facilities to Indian students
to detect influenza viral RNA or nucleic
within the country and enhance general
acids in respiratory specimens with high
level of education of the country.
sensitivity and high specificity. Notably,
The salient features are available in the the detection of influenza viral RNA or
GS


UGC Guidelines and the UGC Regulations nucleic acids by molecular assays does not
under which greater autonomy viz. necessarily indicate detection of viable virus
or on-going influenza viral replication.
 to admit foreign students up to 30% of
admitted students;  Sputum smear microscopy: This
technique requires the examination of
 to recruit foreign faculty upto 25% of sputum samples using a microscope to
faculty strength; determine the presence of bacteria. In the
 to offer online courses upto 20% of its current case definitions recommended by
programmes; WHO, one positive result is required for a
diagnosis of smear-positive pulmonary TB.
 to enter into academic collaboration with
top 500 in the world ranking Institutions  Culture-based methods: A culture test
involves studying bacteria by growing the
without permission of UGC;
bacteria on different substances. This is to
 free to fix and charge fees from foreign find out if particular bacteria are present.
students without restriction; In the case of the TB culture test the test
is to see if the TB bacteria Mycobacterium
 flexibility of course structure in terms of
tuberculosis, are present.
number of credit hours and years to take
a degree;  Tuberculosis (TB) is an infectious disease
usually caused by Mycobacterium
 Complete flexibility in fixing of tuberculosis (MTB) Bacteria.
curriculum and syllabus, etc. has been Tuberculosis generally affects the lungs,
provided to IoEs. but can also affect other parts of the body.

PTS2020/PPP-16/112019/26 8
 India is the country with the highest Supplementary notes:
burden of TB According to the World
Health Organization (WHO) India has National Electronics Policy (NEP)
close to 27 lakh TB cases every year and  National Electronics Policy (NEP) is
4.23 lakh cases die (2016). launched in 2019 and replaced the earlier
 National Strategic Plan (NSP) for 2017- National Electronics Policy, 2012. It
2025 has been set up in line with other aims at making India a global hub
health sector strategies and global efforts, for Electronics System Design and
such as the draft National Health Policy Manufacturing (ESDM) by creating an
2015, World Health Organization’s (WHO) enabling environment for the industry
End TB Strategy and the Sustainable to compete globally. It promotes domestic
Development Goals (SDGs) of the United manufacturing in the entire value-chain
Nations (UN) for TB elimination by 2025. of ESDM, including core components and
 The Health Ministry has signed a materials to increase the domestic value
Memorandum of Understanding (MoU) addition and reduce dependence on import
with Wadhwani Institute for Artificial of electronic goods by focusing on scale, skill
Intelligence (AI) to explore the application and technology.
of Artificial Intelligence technology in its  NEP 2019 primarily targets increasing the
fight against tuberculosis (TB) in 2019.
electronic goods exports. The rising trend

RE
 Revised National TB Programme has been seen in electronics exports amid
(RNTCP) aims at adoption of AI slowdown in Indian economy. The total
technologies for building easy TB combats. value of production of electronic goods
RNTCP is the state-run tuberculosis has increased from $31.2 billion in FY15
(TB) control initiative of the Government to $65.5 billion in FY19. Electronics
of India. exports are led by mobile phones. India
 As per the National Strategic Plan 2012–
O has become the 2nd largest producer
17, the program has a vision of achieving of mobile phones, replacing Vietnam.
a “TB free India”, and aims to achieve India started to become an alternate
Universal Access to TB control services. production destination because of: 1).
Pull Factors include potential domestic
 As part of the collaboration, Wadhwani AI demand and government policies to boost
would support the National TB programme
SC
electronic exports. 2). Push Factors include
by helping it become AI-ready which includes
trade tensions between the US and China.
developing, piloting, and deploying AI-based
This is expected that the positive trend in
solutions. It would support the programme
in vulnerability and hot-spot mapping, India’s electronic exports to continue.
modeling novel methods of screening and  Modified Special Incentive Package
diagnostics. Scheme (M-SIPS): M-SIPS was launched
 India plans to end TB by 2025, five in 2012 under NEP, 2012 and ended in
years ahead of the Global Sustainable 2018.
GS

Development targets. Meanwhile, World


Health Organization (WHO) stated that
countries aiming to reduce their TB burden 21. Correct Option: (b)
by 2035. Explanation:
 Most of the tests used for early detection  Option (b) is correct: Exercise
of TB are not promising. For example, TSENTR2019 is part of the annual series
sputum testing is only 50% accurate and
of large scale exercises that form part of
frequently misses the disease in its early
the Russian Armed Forces’ annual training
stages. Molecular testing – while highly
cycle.
accurate – is too expensive for population
screening in most regions. AI is very helpful Supplementary notes:
in detection of TB with accuracy and
precision. EXTSENTR
 MDR-TB is resistant to anti-TB drugs like  Russia holds this major military exercise
isoniazid and rifampicin every year at one of its four military
Commands i.e Vostok (East), Zapad (West),
TSENTR (Centre) and Kavkas (South).
20. Correct Option: (b) The series of exercise rotates through the
Explanation: four main Russian operational strategic
commands.
 Statement 3 is incorrect: M-SIPS was
launched in 2012 under NEP, 2012 and  These annual exercises have slowly begun
ended in 2018. taking an international character, with

9 PTS2020/PPP-16/112019/26
Belarus participating in ZAPAD-2017 Organization—when its sustained wind
and China and Mongolia participating in speeds top 39 miles (63 km) an hour.
VOSTOK-2018.
 Hurricanes are enormous heat engines that
 This year’s Exercise TSENTR 2019 will be deliver energy on a staggering scale. They
conducted by Central Military Commission draw heat from warm, moist ocean air and
of Russia. Apart from host Russia, military release it through condensation of water
contingents from China, India, Kazakhstan, vapor in thunderstorms.
Kyrgyzstan, Tajikistan, Pakistan and
Uzbekistan will also take part in this mega  Hurricanes spin around a low-pressure
event. center known as the eye. Sinking air makes
this 20- to 40-mile-wide (32- to 64-kilometer-
 The exercise aims at evolving drills of the wide) area notoriously calm. But the eye is
participating armies and practicing them in surrounded by a circular “eye wall” that
the fight against the scourge of international contains the storm’s strongest winds and
terrorism thereby ensuring military security rain.
in the strategic central Asian region.
Hurricane Dorian
 The TSENTR-2019 strategic measures
will focus on evaluating the level of troop  Dorian is estimated to be the second-most-
preparedness, the acquisition of the powerful hurricane ever recorded in the

RE
required skills and raising the level of inter- Atlantic Ocean and ties the record for the
operability and demonstrate the readiness most-powerful storm to make landfall,
of the participating armies. according to the National Weather Service
 The storm is not currently expected to make
22. Correct Option: (b) landfall in the US; it should instead stay
uncomfortably close offshore.
Explanation:
CO
 The storm could bring several inches of
 Option (b) is correct: The size of
rain or more for parts of Florida and the
Hurricane is mainly measured by the
Southeast.
Saffir-Simpson scale.
 The deadliest aspect of a hurricane tends
Supplementary notes: to be storm surge (flooding caused
Hurricane by seawater pushed onshore by the
hurricane’s winds).
 A hurricane is a large rotating storm with
high speeds of wind that gust at least  Reason behind downgrading of
S

74 mph that forms over warm waters in Category of Dorian: Dorian has slowed
tropical areas. down because a high pressure ridge that
was steering the storm westward has
 Hurricanes have three main parts, the calm
weakened. Now, the storm is essentially
eye in the center, the eyewall where the
GS

winds and rains are the strongest, and the waiting for another external force before it
rain bands which spin out from the center starts moving quickly again.
and give the storm its size
 In the southern hemisphere, hurricanes 23. Correct Option: (b)
rotate in a clockwise direction, and in
the northern hemisphere they rotate in an
Explanation:
anti-clockwise direction. This is due to  Statement 1 is incorrect: India is a
what’s called the Coriolis Force, produced major part of this project but it is not
by the Earth’s rotation. indigenously designed and developed
How are hurricanes formed? in India.

 Hurricanes begin as tropical disturbances Supplementary notes:


in warm ocean waters with surface
Project Miniature Sun
temperatures of at least 80 degrees
Fahrenheit (26.5 degrees Celsius). Those  Nick named as ‘miniature sun’,
low-pressure systems are fed by energy International Thermonuclear
from warm seas. Experimental Reactor (ITER) is the
 A storm with wind speeds of 38 miles (61 largest plasma based fusion reactor ever
km) an hour or less is classified as a tropical built.
depression. It becomes a tropical storm—and  It is the costliest technological project of the
is given a name, according to conventions 21st century with an estimated construction
determined by the World Meteorological cost of $25 Billion.

PTS2020/PPP-16/112019/26 10
 The project site is located in Cadarache,  The new finding offers an ‘unprecedented
Southern France. opportunity’ to gain insight into the
composition and climate of habitable-zone
 The term ‘Thermonuclear’ indicates the
planets.
nuclear fusion reaction.
 K2-18b is not ‘Earth 2.0’ as it is significantly
 ITER will be two times the size of the largest
heavier and has a different atmospheric
fusion reactor present and the chamber
composition.
volume will be 10 times the present one.
 This study contributes to our understanding
 It is worth mentioning here that the mega
of habitable worlds beyond our Solar
project is undertaken by USA, Russia, South
System and marks a new era in exoplanet
Korea, China, Japan, European Union and
research.
India.
India’s Contribution What Is an Exoplanet?
 India has also provided a Cryostat, the  All of the planets in our solar system orbit
world’s largest refrigerator, weighing around the Sun. Planets that orbit around
around 3800 tons and made with stainless other stars are called exoplanets.
steel. It is built by Larsen & Toubro  Exoplanets are very hard to see directly with
telescopes. They are hidden by the bright glare

RE
 It will cover the entire structure and
keep the magnetic components at a very of the stars they orbit.
low temperature (less than -200OC) for  There are many methods of detecting
maintaining the superconductivity of exoplanets. Transit photometry and Doppler
magnets. spectroscopy have found the most, but these
methods suffer from a clear observational bias
favoring the detection of planets near the star;
24. Correct Option: (b) O thus, 85% of the exoplanets detected are inside
Explanation: the tidal locking zone.

 Statement 3 is incorrect: It is eight times  One way to search for exoplanets is to look for
the mass of Earth and not Jupiter. “wobbly” stars. A star that has planets doesn’t
orbit perfectly around its center. From far
SC
Supplementary notes: away, this off-center orbit makes the star look
like its wobbling.
K2-18b
 The most massive planet listed on the NASA
 K2-18b an exoplanet that orbits around Exoplanet Archive is HR 2562 b, about 30
a small red dwarf star K2-18, nearly 110 times the mass of Jupiter.
light-years away in the constellation Leo.
K2-18b was first discovered in 2015 by

25. Correct Option: (d)
GS

Nasa’s now retired Kepler space telescope.


 It was found orbiting within the red dwarf Explanation:
star’s habitable zone, making it an ideal  All statements are correct
candidate to have liquid surface water.
Supplementary notes:
 Having a mass that is eight times greater
than Earth’s, K2-18b is also known as eSIM
Super-Earth exoplanets with masses
 An eSIM is exactly what it sounds like an
between those of Earth and Neptune.
electronic, or embedded, SIM. Instead of a
 The Kepler satellite mission discovered physical card, SIM technology is built right
nearly two-third of all known exoplanets. into your phone. It’s a small chip that’s
While the mission indicated that five to 20 used to authenticate our identity with our
per cent of these are located in the habitable carrier.
zone of their stars.
 One of the advantages of eSIM technology
 Water vapour has been detected on K2-18b is that it makes it much easier to switch
a potentially ‘habitable’ planet by Nasa and carriers. Instead of having to order a new
the European space agency’s Hubble Space SIM and wait around for it to arrive, you
Telescope. can switch to a new carrier straight from
your phone.
 K2-18b could be the only exoplanet known
to have both water and temperatures in its  eSIM technology supports multiple
atmosphere to sustain liquid water on a accounts and switching between them
rocky surface is super easy.

11 PTS2020/PPP-16/112019/26
 It will help make devices smaller. 28. Correct Option: (b)
 It could be extremely helpful for Explanation:
wearables devices.
 Statement 1 is incorrect: The Bonn
Challenge is not a new global commitment
26. Correct Option: (c) but rather a practical means of realizing
many existing international commitments,
Explanation: including the CBD Aichi Target 15.
 Both statements are correct
Supplementary notes:
Supplementary notes:
Bonn Challenge
Uyghur Community  The Bonn Challenge is a global effort to bring
 The Uyghurs are Turkic-speaking 150 million hectares of the world’s deforested
Muslims from the Central Asian and degraded land into restoration by 2020,
region. and 350 million hectares by 2030.
 The largest population lives in China’s  It was launched in 2011 by the Government
autonomous Xinjiang region, in the of Germany and IUCN, and later endorsed
country’s north-west. and extended by the New York Declaration

RE
on Forests at the 2014 UN Climate
 The Uyghurs are one of a number of Summit.
persecuted Muslim minorities in Xinjiang,
including the Kazakhs, Uzbeks, Tajiks,  Bonn Challenge is the forest landscape
Kyrgyz and Hui. restoration (FLR) approach, which
aims to restore ecological integrity at
 Many Uyghur communities also live in the same time as improving human
countries neighbouring China, such as well-being through multifunctional
Uzbekistan, Kyrgyzstan and Kazakhstan, landscapes.
CO
Australia.
 The Bonn Challenge is not a new global
commitment but rather a practical
27. Correct Option: (a) means of realizing many existing
international commitments, including
Explanation: the CBD Aichi Target 15, the UNFCCC
 Option (a) is correct: It was the largest REDD+ goal, and the Rio+20 land
flying animal which was a plane-sized degradation neutrality goal.
reptile.
S

Supplementary notes: 29. Correct Option: (d)


Cryodrakon Boreas Explanation:
GS

 New species of pterosaur, the plane-sized  Statement 1 is incorrect: The “Ban


reptiles that lorded over primeval skies Amendment” provides for the prohibition
above T-rex, Triceratops and other dinosaurs by Parties listed in Annex VII (members
of the late Cretaceous. of OECD, EU, Liechtenstein) of all
transboundary movements of hazardous
 With a wingspan of 10 m and weighing wastes including electronic wastes and
250 kg, Cryodrakon boreas rivals another obsolete ships which are destined for final
pterosaur as the largest flying animal of all disposal operations from OECD to non-
time OECD States.
 Its remains were first discovered more  Statement 2 is incorrect: India has not
than 30 years ago in Alberta, Canada, yet ratified the Basel ban amendment.
elicited scant excitement because of the
misclassification. Supplementary notes:
 Like other winged reptiles living at the Basel Convention
same time, about 77 million years ago, C.
boreas was carnivorous and probably fed  The Basel Convention on the Control of
on lizards, small mammals and even baby Transboundary Movements of Hazardous
dinosaurs. Wastes and their Disposal was adopted
on 22 March 1989 by the Conference of
 Despite their large size and wide distribution Plenipotentiaries in Basel, Switzerland, in
across North and South America, Asia, response to a public outcry following the
Africa and Europe only fragmentary discovery, in the 1980s, in Africa and other
remains have been unearthed, making the parts of the developing world of deposits of
new find especially important. toxic wastes imported from abroad.

PTS2020/PPP-16/112019/26 12
 It does not address the movement of  Statement 3 is incorrect: ASF is not
radioactive waste. a risk to human health as it is relatively
harmless.
 To implement and restrict the trade of
hazardous waste between more developed Supplementary notes:
countries and less developed countries an
organization is formed which is known as What is African swine fever?
Basel Action Network (BAN).
 Historically, outbreaks have been reported
The provisions of the Convention center in Africa and parts of Europe, South
around the following principal aims America, and the Caribbean. More recently
(since 2007) the disease has been reported
 The reduction of hazardous waste generation
in multiple countries across Africa, Asia and
and the promotion of environmentally sound
Europe, in both domestic and wild pigs.
management of hazardous wastes, wherever
the place of disposal.  African swine fever (ASF) is a highly
 The restriction of transboundary movements contagious haemorrhagic viral disease
of hazardous wastes except where it is (severe illness, sometimes associated
perceived to be in accordance with the with bleeding), of domestic and wild
principles of environmentally sound pigs.
management. It is caused by a large DNA virus of the

RE

 A regulatory system applying to cases where Asfarviridae family, which also infects ticks
transboundary movements are permissible. of the genus Ornithodoros.

The Basel Convention BAN Amendment Transmission and spread:


 The Ban Amendment was originally adopted  The epidemiology of ASF is complex and
as a decision of the second meeting of the varies depending on the environment, types
Conference of the Parties in March 1994.
O of pig production systems, the presence/
absence of competent tick vectors, human
 According to amendment, it was agreed
that such “Annex VII Parties” prohibit and behaviour, and the presence/absence of wild
phase out all transboundary movements of pigs.
hazardous wastes destined for recovery or  Direct contact with infected domestic or wild
SC
recycling operations from OECD to non- pigs: This transboundary animal disease
OECD States by 31 December 1997. (TAD) can be spread by live or dead pigs,
 At COP-3 in 1995, Parties adopted the same domestic or wild, and pork products.
as a further amendment to the Convention  Indirect contact, through ingestion of
known as the “Ban Amendment”. contaminated material (e.g. food waste,
 The Ban Amendment had been stalled for feed, or garbage). Contaminated fomites,
all these years till now due to uncertainty or biological vectors (soft ticks of the genus
over how to interpret the Convention. Ornithodoros) where present.
GS

Ratification status of other major Prevention and control:


countries
 Currently there is no approved vaccine for
 According to BAN the United States, the ASF.
world’s most wasteful country per-capita,
has not ratified the Basel Convention, nor  United Nations Food and Agriculture
the Ban Amendment Organization (FAO) reported that ASF has
caused the deaths of more than 3.7 million
 Other developed countries like Canada, pigs across a vast swathe of Asia, primarily
Japan, Australia, and New Zealand, in its east and south-east, where pork is the
likewise, have e-waste export problems and primary meat staple.
they too have so far refused to ratify the
Ban Amendment.  Although signs of ASF and classical
swine fever (CSF) may be similar,
 South Korea, Russia, India, Brazil, and
the ASF virus is unrelated to the CSF
Mexico are yet to ratify the ban.
virus.
 ASF is not a risk to human health as it
30. Correct Option: (a) is relatively harmless.
Explanation:
 Statement 2 is incorrect: Although signs 31. Correct Option: (d)
of ASF and classical swine fever (CSF) may
be similar, the ASF virus is not related to Explanation:
the CSF virus.  All pairs are correctly matched

13 PTS2020/PPP-16/112019/26
Supplementary notes:
 Crimean-Congo
Meaning of Vectors haemorrhagic fever

 Vectors are living organisms that can  Lyme disease


transmit infectious diseases between Ticks  Relapsing fever
humans or from animals to humans. (borreliosis)
 Many of these vectors are bloodsucking  Tick-borne encephalitis
insects, which ingest disease-producing
 Tularaemia
microorganisms during a blood meal from
an infected host (human or animal) and Triatomine  Chagas disease (American
later inject it into a new host during their bugs trypanosomiasis)
subsequent blood meal.  Sleeping sickness (African
Tsetse flies
 Mosquitoes are the best known disease trypanosomiasis)
vector. Others include ticks, flies, sandflies,  Plague (transmitted by
fleas, triatomine bugs and some freshwater fleas from rats to humans)
Fleas
aquatic snails.
 Rickettsiosis
 Meaning of Vector-borne diseases:

RE
 Onchocerciasis (river
Vector-borne diseases are human illnesses Black flies
 blindness)
caused by parasites, viruses and bacteria
that are transmitted by vectors.  Typhus and louse-borne
Lice
relapsing fever
 The major vector-borne diseases, together,
account for around 17% of all infectious
diseases. The burden of these diseases is 32. Correct Option: (b)
CO
highest in tropical and subtropical areas
Explanation:
and they disproportionately affect the
poorest populations.  Statement 1 is incorrect: The PM-KMY is
a Central Sector Scheme, administered by
 Since 2014, major outbreaks of dengue, the Department of Agriculture, Cooperation
malaria, chikungunya, yellow fever and & Farmers Welfare, Ministry of Agriculture
Zika have afflicted populations, claimed & Farmers’ Welfare.
lives and overwhelmed health systems in
Supplementary notes:
many countries.
S

Pradhan Mantri Kisan Maan-Dhan


Main vectors and diseases they transmit: Yojana
Vector Disease  The PM-KMY is a Central Sector Scheme,
GS

administered by the Department of


 Chikungunya Agriculture, Cooperation & Farmers
 Dengue fever Welfare, Ministry of Agriculture & Farmers’
Welfare.
Aedes  Lymphatic filariasis
 The Pradhan Mantri Kisan Maan-Dhan
mosquit Rift Valley fever
 Yojana (PM-KMY) provides for an assured
 Yellow fever monthly pension of Rs. 3000/- to all land
holding Small and Marginal Farmers
 Zika (SMFs), whether male or female, on their
attaining the age of 60 years.
 Malaria
 The amount of the monthly contribution
 Lymphatic filariasis
ranges between Rs.55 to Rs.200 per month
Anopheles  Culex mosquito depending upon the age of entry of the
mosquit farmers into the Scheme.
 Japanese encephalitis
 The Central Government will also make an
 Lymphatic filariasis equal contribution of the same amount in
 West Nile fever the pension fund.

 Leishmaniasis  The spouse is also eligible to get a separate


(Kala-Azar) pension of Rs.3000/- upon making separate
Sandflies contributions to the Fund.
 Sandfly fever (phelebotomus
fever)  In case of death of the farmer before
retirement date, the spouse may continue

PTS2020/PPP-16/112019/26 14
in the scheme by paying the remaining through an Integrated Teacher
contributions till the remaining age of the Training Programme called NISHTHA
deceased farmer. is launched by The Department of
School Education and Literacy under
 If the farmer dies after the retirement date,
the Centrally Sponsored Scheme of
the spouse will receive 50% of the pension
Samagra Shiksha in 2109-20.
as Family Pension.
 NISHTHA is a capacity building programme
 After the death of both the farmer and the
spouse, the accumulated corpus shall be for “Improving Quality of School Education
credited back to the Pension Fund. through Integrated Teacher Training”.
 It aims to build competencies among all
Eligibility
the teachers and school principals at the
 All Small and Marginal Farmers (SMFs) elementary stage.
in all States and Union Territories of the
 NISHTHA is the world’s largest teachers’
country, who are of the age of 18 years
training programme of its kind.
and above and upto the age of 40 years,
and who do not fall within the purview of  The basic objective of this massive training
the exclusion criteria as mentioned in the programme is to motivate and equip
guidelines, are eligible to avail the benefits teachers to encourage and foster critical
of this Scheme by joining it. thinking in students. The initiative is first

RE
 Farmers falling within the purview of the of its kind wherein standardized training
exclusion criteria are not eligible for the modules are developed at national level for
benefit. all States and UTs.

Exclusion criteria  This integrated programme aims to build


the capacities of around 42 lakh participants
 SMFs covered under any other statuary covering all teachers and Heads of Schools
social security schemes such as National at the elementary level in all Government
Pension Scheme (NPS), Employees’ State
O
Insurance Corporation scheme, Employees’
Fund Organization Scheme etc.
schools, faculty members of State Councils
of Educational Research and Training
(SCERTs), District Institutes of Education
 Farmers who have opted for Pradhan Mantri and Training (DIETs) as well as Block
SC
Shram Yogi Maan Dhan Yojana (PMSYM) Resource Coordinators and Cluster Resource
administered by the Ministry of Labour & Coordinators in all States and UTs.
Employment.  Training will be conducted directly by
 Farmers who have opted for Pradhan 33120 Key Resource Persons (KRPs) and
Mantri Laghu Vyapari Maan-dhan Yojana State Resource Persons (SRP) identified
(PM-LVM) administered by the Ministry of by the State and UTs, who will in turn
Labour & Employment. be trained by 120 National Resource
Persons identified from National Council
GS

 The following categories of beneficiaries of of Educational Research and Training


higher economic status are not eligible for (NCERT), National Institute of Educational
benefits under the scheme: Planning and Administration (NIEPA),
 All Institutional Land holders. Kendriya Vidyalaya Sangathan (KVS),
Navodaya Vidyalaya Samiti (NVS), Central
 Former and present holders of constitutional Board of Secondary Education (CBSE) and
posts
Non-Government Organisations.
 Professionals like Doctors, Engineers,
 The prominent features of this integrated
Lawyers, Chartered Accountants, and
programme are activity based modules
Architects registered with Professional
including educational games and quizzes,
bodies and carrying out profession by
Social-emotional learning, motivational
undertaking practice.
interactions, team building, preparation for
school based assessment, in-built continuous
33. Correct Option: (b) feedback mechanism, online monitoring and
support system, training need and impact
Explanation: analysis (Pre and Post training).
 Option (b) is correct
Supplementary notes: 34. Correct Option: (c)
NISHTHA Explanation:
 National Mission to improve learning  Statement 2 is incorrect: It is issued by
outcomes at the elementary level Government of India.

15 PTS2020/PPP-16/112019/26
Supplementary notes: Supplementary notes:
ILP Mietei Community
 ILP is an official travel document issued  Mietei community of Manipur who are
by the Government of India to grant largely Hindus are believed to be behind
inward travel of an Indian citizen into the violent protests. The tribal populations
a protected area for a limited period. in Manipur have no role in protests.
Visitors are not allowed to purchase  Meiteis demand the Scheduled Tribe status
property in these regions. and the tribals are saying if Meiteis are
given the ST status they will buy all their
 Validity of permit is generally 15-20 days. lands.
 Prior to Manipur’s merger with India,  The influx of foreign tourists and illegal
Manipur had become a state and nearly immigration from Bangladesh, Nepal and
had the ILP system. However, it was only Myanmar has contributed to the crisis.
in 1951 when the then chief commissioner This has created fear among the locals over
Himmat Singh lifted the rule and allowed employment and availability of resources.
unregulated entry of outsiders into the Meiteis want strict imposition of ILP
region. in Manipur to protect their culture and
identity.

RE
 According to the 2011 census, the population
of Manipur is a little over 27 lakh. Of this,  Meiteis abandoned their traditional
only 17 lakh (1.7 million) are indigenous Sanamahi faith and converted en mass to
people and 10 lakh (one million) are Hinduism.
outsiders.
 The ongoing agitation took an ugly turn 36. Correct Option: (d)
when villagers torched the houses of MP Explanation:
CO
and MLAs.
 All statements are correct:
 Currently, the Inner Line Permit is
operational in Arunachal Pradesh, Supplementary notes:
Mizoram and Nagaland. In addition to Market Intervention Scheme
these, Meghalaya, Manipur, Andaman
and Nicobar Islands and Leh also  Jammu and Kashmir Governor, Satya Pal
require Indian citizens to have a permit Malik launched market intervention scheme
for apples of J&K.
before they enter.
S

 Kashmir’s apple is battling to get exported


 It has been issued under the Bengal outside the State this year due to shut
Eastern Frontier Regulation, 1873 and the down and suspected militants campaigning
conditions and restrictions vary from state against the fruit’s export.
to state.
GS

 In the absence of MSP like scheme


 It can be issued for travel purposes solely. for apples J&K Govt. plans to procure
12 lakh metric tonnes worth of 8,000
 Visitors are not allowed to purchase
crores under market intervention
property in these regions. However, there
scheme with the help of the National
might be a different set of rules for long Agriculture Cooperative Marketing
term visitors, though they are not valid for Federation of India (NAFED). It makes
central government employees and security 60% of the estimated annual apple
forces. production.
Purpose  It has been approved by the Department
of Agriculture and Cooperation, Union
 To regulate the movement of visitors into Ministry of Agriculture under the aegis
sensitive regions of Ministry of Home Affairs, for the
 To strengthen security of the sensitive areas current apple harvesting season.
near to international borders  In J&K, the scheme will be implemented
by the Directorate of Planning and
 To protect the cultural ethos of a specific
Marketing, supported by the Directorate
region
of Horticulture (Kashmir) and Jammu and
Kashmir Horticultural Produce, Marketing
35. Correct Option: (c) and Processing Corporation Ltd.
 Kashmir produced 20 lakh metric tonnes of
Explanation: apple in 2018, contributing 73% to the total
 Both statements are correct apple production of the country.

PTS2020/PPP-16/112019/26 16
 The period of procurement would be  The fact that the SCO also strives
for six months, starting September, at to strengthen mutual trust and good
a cost of Rs. 8,000 crore. neighbourly relations between its member
states.
 Apples to be bought from the doorsteps
of the orchardists. Direct payment  On 9 June 2017, at a summit in Astana,
will be made to their account. India and Pakistan officially joined SCO as
Transportation will be the job of the full members.
procurer only.
 The government has set up mandis for the 38. Correct Option: (d)
scheme at Srinagar’s Parimpora, Baramulla’s
Sopore, Shopian and Anantnag’s Botengu. Explanation:
 Option (d) is correct: A gig economy is
Significance
a free market system in which temporary
 It would help stabilize the price of positions are common and organizations
apples and help maintain a uniform contract with independent workers for
rate in the open market across the short-term engagements
country, after India raised tariffs on
Supplementary notes:
apple imports from the United States

RE
earlier this year. Gig Economy
 It will provide the much needed support to  In a gig economy, temporary, flexible jobs
the fruit growers in Jammu and Kashmir. are commonplace and companies tend
Earlier, government failed to provide the toward hiring independent contractors and
required number of trucks for transportation freelancers instead of full-time employees.
of the apples.
 A gig economy undermines the traditional
economy of full-time workers who rarely



from the doorsteps of the farmers.
O
Ease and convenience for the farmers as
apples will be procured and transported

It will help in building the necessary 


change positions and instead focus on a
lifetime career.
The gig economy can benefit workers,
businesses, and consumers by making work
SC
forwards and backward linkage in the apple
industry in Jammu and Kashmir. The more adaptable to the needs of the moment
building of cold storages will also receive and demand for flexible lifestyles.
the necessary attention in the region.  At the same time, the gig economy can have
downsides due to the erosion of traditional
economic relationships between workers,
37. Correct Option: (a) businesses, and clients.
Explanation:
GS

 Statement 3 is not correct: India is not 39. Correct Option: (b)


founding member of Shanghai Cooperation
Organisation. Explanation:
 Option (b) is correct: Logistics
Supplementary notes: Performance Index (LPI) is published by
Shanghai Cooperation Organisation World Bank.
(SCO) Supplementary notes:
 The SCO, an intergovernmental body for
Logistics Performance Index
security and economic cooperation in the
Eurasian region, was formed in 2001 by  The Logistics Performance Index (LPI) is an
the ‘Shanghai Five’ (China, Kazakhstan, interactive benchmarking tool created by
Kyrgyzstan, Russia and Tajikistan), in the World Bank to help countries identify
the wake of the Soviet Union’s collapse in the challenges and opportunities they face
1991. in their performance on trade logistics
and what they can do to improve their
 Uzbekistan joined the SCO in 2001, with performance.
India and Pakistan following suit in 2017.
 The LPI is based on a worldwide survey
 It aimed to cooperate against non- of operators on the ground (global freight
traditional security challenges, in forwarders and express carriers), providing
particular fighting the so-called “three feedback on the logistics “friendliness” of
evils” of terrorism, separatism, and the countries in which they operate and
extremism. those with which they trade.

17 PTS2020/PPP-16/112019/26
 They combine in-depth knowledge of the untouchability in the society by the higher
countries in which they operate with caste people for lower caste people.
informed qualitative assessments of other
countries where they trade and experience  He spread a message that “God created
of global logistics environment. man and not man created God” means
everyone is created by the God and has
 Feedback from operators is supplemented equal rights on this earth.
with quantitative data on the performance
of key components of the logistics chain in
the country of work. 41. Correct Option: (a)
Explanation:
40. Correct Option: (d)
 Statement 3 is incorrect: Vultures are
Explanation: native to India, Pakistan and Nepal
 All statements are correct Supplementary notes:
Supplementary notes Critically Endangered Indian Vulture
Sant Ravidas—the mystic Guru of Bhakti (Gyps indicus)
movement  Nine species of vulture can be found living
in India, but most are now in danger of

RE
 Guru Ravidas was a famous saint of
the Bhakti Movement and was born in extinction.
1377 C.E. at Mandhuadhe which is located  The number declined from 40 million in the
in Uttar Pradesh, India. 80s to a few thousand by 2009.
 He was one of the most famous and leading  The major reason behind the vulture
star of the nirgunasampradaya means population getting nearly wiped out was
santparampara and lead the North Indian the drug Diclofenac, found in the carcass of
Bhakti movement.
CO
cattle the vultures fed on. The drug, whose
 He has given variety of spiritual and social veterinary use was banned in 2008, was
messages through his great writings of commonly administered to cattle to treat
poetry to his lovers, followers, community inflammation.
people, and society people to reform their
mind and show their boundless love towards  The IUCN Red Data Book has listed Gyps
God. bengalensis as “critically endangered”.

 He was the vision of people as a messiah in  The dramatic vulture decline observed
order to complete the social and spiritual across India presents a range of ecological
S

needs. He was the spiritually rich person threats, by influencing the numbers and
worshipped by the people. distribution of other scavenging species.
Increased feral dog populations have been
 Guru Ravidas was also known as Raidas, reported all over in India, posing many
GS

Rohidas or Ruhidas. associated disease risks such as rabies to


 Sant Guru Ravidasji is considered as a humans and wildlife.
spiritual Guru of the Meera Bai
 The Hindu religion, which represents 80%
 She was very impressed by the teachings of the country’s population, is particularly
of Guru RavidassJi and became the great favourable to vultures. Cows are considered
follower of him. sacred by majority of Hindus, so cow meat
 Meera Bai has written some lines in is generally not consumed. This results in
the respect of his Guru “Guru Milyaa the cow’s corpse being left to be fed on by
Ravidasji” vultures.

 He was sent by the God on the earth when FACT BOOK


required him to save the real Dharma as at
that time social and religious patterns were  Habitat and Ecology: Forest, Grassland,
distressing because of the lots of man-made Shrubland, Savanna, Artificial/Terrestrial
discriminations over social beliefs, caste,  IUCN status: Critically Endangered
colour and etc.
 Major Threat: Diclofenac Drug
 He bravely faced all the discriminations and
answered people about the real definition of  Vulture Care Centre (VCC): set up at
beliefs and castes. Pinjore, Haryana.
 He taught people that one is not known by  Vulture Conservation and Breeding
his caste, religion or believes for God, he Centres (VCBC) in India: 9
known only for his great actions (or karma).
 Extant (resident): India; Pakistan
He also worked against the system of

PTS2020/PPP-16/112019/26 18
42. Correct Option: (c)  Section 4(2) of the Right to Information
(RTI) Act of 2005 requires public authorities
Explanation: to disclose information pro-actively. The
 Both statements are correct Jan Soochna Portal will not require
citizens to submit RTI application for
Supplementary notes: seeking information.
HOG Technology  Aim: To provide information on government
 The new technology is expected to transform services and authorities to public suo-moto
the way air conditioners (ACs) run and in the true spirit of right to information.
power is supplied in the railway coaches
 The portal will prevent anomalies and
 In HOG technology, power is fed from the corruption in government.
electric locomotive to the train to cater for
the Hotel Load of the train.  A mobile app will be developed and self-
service portals will be installed in villages
 It will reduce the power consumption so that people can access the Jan Soochna
to a drastic level thus opening the door
Portal easily and get the information they
towards sustainable use of resources.
require.

43. Correct Option: (a)

RE
45. Correct Option: (c)
Explanation:
Explanation:
 Option (a) is correct
 Both statements are correct
Supplementary notes:
Supplementary notes:
Rome Statute
 The Rome Statute, a multilateral treaty,
O NIRVIK
is the foundation and governing document  Export Credit Guarantee Corporation
of the International Criminal Court of India (ECGC) has introduced
(ICC). ‘NIRVIK’ scheme to ease the lending
 The Rome Statute of the International process and enhance loan availability
SC
Criminal Court is an agreement that led to for exporters.
the formation of the International Criminal
Court (ICC).  Insurance cover guaranteed will cover
up to 90 percent of the principal and
International Criminal Court interest.
 The International Criminal Court is an  The increased cover will ensure that foreign
intergovernmental organization and and rupee export credit interest rates
international tribunal that sits in The Hague, are below 4 percent and 8 percent
Netherlands.
GS

respectively for the exporters.


 The ICC has jurisdiction to prosecute individuals
 The insurance cover will include both pre
for the international crimes of genocide, crimes
against humanity, war crimes, and crimes of and post-shipment credit.
aggression.  The gems, jeweller and diamond (GJD)
 It is intended to complement existing national sector borrowers with limit of over
judicial systems and it may therefore exercise Rs 80 crore will have a higher premium
its jurisdiction only when certain conditions rate in comparison to the non-GJD sector
are met, such as when national courts are borrowers of this category due to the higher
unwilling or unable to prosecute criminals or loss ratio.
when the United Nations Security Council or
individual states refer situations to the Court.  For accounts with limits below Rs 80 crore,
the premium rates will be moderated to
0.60 per annum and for those exceeding Rs
44. Correct Option: (a) 80 crore, the rates will be 0.72 per annum
Explanation: for the same enhanced cover.

 All statements are correct  It mandates inspection of bank documents


and records by ECGC officials for losses
Supplementary notes: exceeding Rs. 10 crore as against the present
Jan Soochna Portal Rs 1 crore.

 Jan Soochna Portal is the first ever public  The banks shall pay a premium to ECGC
information portal launched by the monthly on the principal and interest as the
government of Rajasthan. cover is offered for both outstanding.

19 PTS2020/PPP-16/112019/26
 The Export Credit Guarantee weeks we can pick up lots of information on
Corporation of India (ECGC) is a fully structural problems.
government-owned company that
 Invasive Tests: These test the amniotic
was established in 1957 to promote
fluid, chorionic villus tissue or foetal blood.
exports by providing credit insurance
Sometimes we even do a skin biopsy of the
services.
baby. The other tests will give you a probable
 The ECGC provides Export Credit Insurance diagnosis – each one has its own proportion
to Banks (ECIB) to protect the banks from of false positives and false negatives. But
losses on account of export credit at the Pre when we test the tissue, we are actually
and Post-Shipment stage given to exporters looking at the genes, chromosomes, DNA,
due to the risks of insolvency or protracted and this gives us a confirmatory result for
default of the exporter borrower. certain genetic diseases. Amniocentesis is
an example of invasive test.

46. Correct Option: (c)


48. Correct Option: (c)
Explanation:
Explanation:
 Both statements are correct
 Both statements are correct
Supplementary notes:

RE
Supplementary notes:
Teaser Loans
About LRO:
 Teaser Loans is a term used
for loans that offer low, fixed interest  Lunar Reconnaissance Orbiter (LRO)
rates in the initial few years, but switch is a robotic mission that set out to map
to floating rates thereafter. the moon’s surface and, after a year of
exploration, was extended with a unique set
 SBI first introduced teaser loan in
CO
of science objectives.
2009-10.
 LRO entered lunar orbit on June 23, 2009
 The RBI is not likely to allow the return of
and after spacecraft commissioning, the
teaser home loans, following the introduction
Exploration Mission began on September
of the external benchmark-linked loan
15, 2009.
pricing.
 The Exploration Mission completed on
 SBI had tried to bring back teaser
September 15, 2010 when responsibility
loans several times in the past. However,
for LRO was transferred to NASA’s Science
the RBI does not seem to be in its favour.
S

Mission Directorate for a two-year Science


 The RBI felt banks could use the low interest Mission with a new set of science goals. The
rates to lure customers into taking these LRO mission has been extended to continue
loans and then move to a higher market lunar science and exploration.
GS

rate later. In 2011, the central bank had


increased the provisioning on these loans to Focus of the exploration Mission:
discourage banks from offering them.  The Exploration Mission was focused
on supporting the extension of human
presence in the solar system.
47. Correct Option: (c)
 LRO continues to help identify sites close to
Explanation: potential resources with high scientific value,
 Both statement are correct favourable terrain and the environment
necessary for safe future robotic and human
Supplementary notes: lunar missions.
Various Prenatal Diagnosis Tests  LRO observations have enabled numerous
ground-breaking discoveries, creating a
 Dual Marker and Triple Marker
new picture of the moon as a dynamic and
Tests: These tests look at the level of
complex body. These developments have set
certain hormones which are ‘markers’
up a scientific framework through which to
associated with Down’s Syndrome.
challenge and improve our understanding
 Ultrasound Tests: They are used of processes throughout the solar system.
extensively, four-five times in the
Some of the LRO exploration and science
pregnancy. Here we look for markers for
results:
certain abnormalities. In the case of Down’s
Syndrome, for example, certain physical  In polar shadowed regions found the coldest
characteristics suggest that the foetus has spots measured (below 30 K) in the solar
this condition. With an ultrasound at 18-19 system.

PTS2020/PPP-16/112019/26 20
 Discovered significant subsurface hydrogen for each quarter. These notes allow foreign
deposits in regions cold enough for water ice high networth individuals, hedge funds
to survive, as well as in additional hydrogen and other investors to put money in Indian
deposits in warmer areas where surface markets without being registered with
water ice is not thermally stable. SEBI, thus making their participation easy
and smooth.
 New (<5 years old) impact craters and are
found to be widespread across the lunar Advantages of participatory notes
surface, with a surprising abundance of
related surface changes.  Anonymity: Any entity investing in
participatory notes is not required to
 Developed an improved catalogue of lunar register with SEBI, whereas all FIIs have to
craters larger than 20 km in diameter, compulsorily get registered. It enables large
thus providing constraints on the ancient hedge funds to carry out their operations
impactor population that affected the inner without disclosing their identity.
solar system.
 Ease of trading: Trading through
 First radar measurements of the lunar far participatory notes is easy because they
side. are like contract notes transferable by
 Characterized relatively young volcanic endorsement and delivery.
complexes, such as Ina, and revealed first Tax saving: Some of the entities route their

RE

direct evidence of the presence of highly investment through participatory notes to
silicic volcanic rocks on the Moon. take advantage of the tax laws of certain
 Measured galactic cosmic ray interactions preferred countries.
with the Moon during a period with the  P-Notes also aid in saving time and costs
largest cosmic ray intensities observed associated with direct registrations.
during the space age.
O Disadvantages of P-Notes
 Created the first cosmic ray albedo
proton map of the Moon.  Because of the anonymous nature of the
instrument, the investors could be beyond
the reach of Indian regulators.
49. Correct Option: (b)
P-Notes are being used in money laundering
SC

Explanation: with wealthy Indians, like the promoters
of companies, using it to bring back
 Option (b) is correct: Participatory Notes unaccounted funds and to manipulate their
are regulated by Security and Exchange stock prices.
Board of India (SEBI).
Supplementary notes: 50. Correct Option: (d)
Participatory Motes (P- Notes)
Explanation:
GS

P-notes are issued by registered foreign  All statements are correct


portfolio investors (FPIs) to overseas investors
who wish to be a part of the Indian stock market Supplementary notes:
without registering themselves directly after
National Pension Scheme for Traders
going through a due diligence process.
and Self Employed Persons
Who issues P-Notes and what is the
 Under this scheme all shopkeepers,
process? retail traders and self-employed
 Participatory notes are issued by persons are assured a minimum
brokers and FIIs registered with SEBI. monthly pension of Rs. 3,000/- month
The investment is made on behalf of after attaining the age of 60 years.
these foreign investors by the already  All small shopkeepers and self-employed
registered brokers in India. persons as well as the retail traders with
 For example, Indian-based brokerages GST turnover below Rs. 1.5 crore and
buy India-based securities and then issue age between 18-40 years, can enrol for
participatory notes to foreign investors. this scheme. The scheme would benefit
Any dividends or capital gains collected more than 3 crore small shopkeepers and
from the underlying securities go back to traders.
the investors.  The scheme is based on self-declaration
 The brokers that issue these notes or trades as no documents are required except
in Indian securities have to mandatorily Aadhaar and bank account. Interested
report their PN issuance status to SEBI persons can enrol themselves through

21 PTS2020/PPP-16/112019/26
more than 3,25,000 Common Service
Border
Centres spread across the country. Security Force
Sundarban India and (BSF) and
 GSTIN is required only for those with 6.
Maitri Bangladesh Border Guard
turnover above Rs. 40 lakhs. Bangladesh
 The Central Government shall give 50 (BDB)
% share of the monthly contribution and 7. Suryakiran India and Nepal Army exercise
remaining 50% contribution shall be made
by the beneficiary. For example if a person Ex Lamitya Indo-Seychelles
8. Army Exercise
VII Joint Training
with age of 29 years contributes Rs. 100/-
month, then the Central Government also Disaster
contributes the equal amount as subsidy Management
9. SAADMEx SAARC
into subscriber’s pension account every Exercise by
month. NDRF

 The beneficiary should not be income tax Exercise IAF's annual Indian Air
10.
Livewire exercise Force
payer and also not a member of EPFO/
ESIC/NPS (Govt.) /PM-SYM. HAND-IN-
11. India & China Army exercise
HAND
 This scheme will target enrolling 25
lakh subscribers in 2019-20 and 2 crore India & Sri

RE
12. SLINEX Naval Exercise
subscribers by 2023-2024. An estimated 3 Lanka
crore Vyaparis in the country are expected
13. Indra India & Russia Army exercise
to be benefitted under the pension scheme.
India &
14. SAMPRITI Army exercise
Bangladesh
51. Correct Option: (b)
Explanation: 52. Correct Option: (d)
CO
 Pair 2 is incorrectly matched: It is the
first bilateral exercise between Japan Air
Explanation:
Self-Defence Force (JASDF) and the Indian  All statements are correct.
Air Force.
Supplementary notes:
Supplementary notes:
International Migrant Stock
Bilateral Exercises
India was the leading country of origin of
“Dharma Guardian”: It is the first bilateral international migrants in 2019 with a 17.5
S


exercise between the Japan Ground Self- million strong Diaspora, according to new
Defense Force (JGSDF) and the Indian estimates released by the United Nations,
Army in the area of counter-terrorism. which said the number of migrants globally
reached an estimated 272 million.
GS

 ‘SHINYUU MAITRI 18’: It is the first


bilateral exercise between Japan Air Self-  The count of the Indian Diaspora has
Defence Force (JASDF) and the Indian Air increased 10% from 15.9 million in 2015,
Force. making it the largest in the world, according
to the UN’s International Migrant Stock
 “Cope India”: It is the India-U.S. bilateral
2019 released on Wednesday.
exercise.
 The data set was compiled by the UN
Other exercises: Department of Economic and Social Affairs
S. Name of Participating Nature of
Division.
No. exercise country exercise  It now comprises 6.4% of the total global
YUDH India & United migrant population.
1. Army exercise
ABHYAS States
 The United Arab Emirates (UAE) was the
India & top destination of Indian migrants followed
2. KUVERIN Army exercise
Maldives by the US, Saudi Arabia, Pakistan and
Oman.
India &
3. AUSINDEX Naval exercise
Australia  Bangladesh was a top source of migrants
to India, followed by Pakistan, Nepal,
MITRA India & Sri
4.
SHAKTHI Lanka
Army exercise Srilanka and China.

Malabar Global data


India & US &
5. Naval Naval Exercise
Exercise
Japan  In 2019, the number of international
migrants worldwide was nearly 272 million,

PTS2020/PPP-16/112019/26 22
up from 221 million in 2010 and 174 million  The latest Scientific Assessment of Ozone
in 2000. Depletion shows that, parts of the ozone
layer have recovered at a rate of 1-3% per
 More than half of all international migrants
lived in Europe (82 million) or Northern decade since 2000.
America (59 million).  At projected rates, Northern Hemisphere
 Women comprised slightly less than and mid-latitude ozone will heal completely
half of all international migrants. The by the 2030s.
share of women in the total number of
international migrants fell from 49.3 %
in 2000 to 48% cent in 2019. 55. Correct Option: (c)
 In 2019, the percentage of females among Explanation:
all international migrants was highest in  Statement 1 is incorrect: The Atlantic
Northern America (51.8 %) and Europe Meridional Overturning Circulation
(51.4 %).
(AMOC) is a large system of ocean currents
 Central and Southern Asia (49.4 %), and that carry warm water from the tropics
Eastern and South-Eastern Asia (49.3 %) northwards into the North Atlantic.
hosted an almost equal proportion of female
and male migrants. Supplementary notes:

RE
 Europe hosts the largest number of About Atlantic Meridional Overturning
international migrants, at 82 million; Circulation (AMOC):
followed by North America at 59 million;
North Africa and Western Asia host around  The Atlantic Meridional Overturning
49 million migrants. Circulation (AMOC) is a large system of
ocean currents that carry warm water
In News - UN’s International Migrant Stock from the tropics northwards into the
2019 has been released. O North Atlantic.
 AMOC ensures the oceans are continually
53. Correct Option: (a) mixed, and heat and energy are distributed
Explanation: around Earth.
SC
 Option (a) is correct: The Houthi takeover
in Yemen, is known as the September 21
Revolution.
Supplementary notes:
September 21 Revolution
 The Houthi takeover in Yemen, is known
as the September 21 Revolution
GS

 It was a gradual armed takeover by the


Houthis and supporters of former Yemeni
President Ali Abdullah Saleh that pushed
the Yemeni government from power.

54. Correct Option: (a) How does the AMOC work?


Explanation:  The AMOC is a large system of ocean
currents, like a conveyor belt, driven by
 Option (a) is correct: The theme for the differences in temperature, salt content and
World Ozone Day 2019: 32 Years and the water’s density.
Healing
 As warm water flows northwards it cools and
Supplementary notes: some evaporation occurs, which increases
Ozone Day the amount of salt. Low temperature and
a high salt content make the water denser,
 The theme for this year celebrates over and this dense water sinks deep into the
three decades of remarkable international ocean.
cooperation to protect the ozone layer and
the climate under the Montreal Protocol.  The cold, dense water slowly spreads
southwards, several kilometres below the
 The Montreal Protocol has led to the
surface. Eventually, it gets pulled back to
phase-out of 99 per cent of ozone-depleting
chemicals in refrigerators, air-conditioners the surface and warms in a process called
and many other products. “upwelling” and the circulation is complete.

23 PTS2020/PPP-16/112019/26
Effect of climate change on the AMOC:  Methane, black carbon and
hydrofluorocarbons (HFCs) are some
 Climate models suggest that the AMOC examples of short lived pollutants.
will weaken over the 21st Century as
greenhouse gases increase. This is because  They are many times more powerful than
as the atmosphere warms, the surface ocean carbon dioxide at warming the planet.
beneath it retains more of its heat.  But because they are short-lived in the
 All these changes make the ocean water atmosphere, preventing emissions can
lighter and so reduce the sinking in the rapidly reduce the rate of warming.
‘conveyor belt’, leading to a weaker AMOC. Approach of CCAC to reduce short-lived
So the AMOC is very likely to weaken, but climate pollutants:
it’s considered very unlikely that large,
rapid changes in the AMOC, as seen in past The Coalition’s partners and initiative
times, will happen in the 21st Century. participants work in cooperation with key
short-lived climate pollutant emitters and other
 A weaker AMOC will bring less warm water stakeholders from around the world to encourage,
northwards, and this will partly offset the enable and catalyse action to reduce emissions.
warming effect of the greenhouse gases over To achieve real and ambitious reductions, the
Western Europe. Coalition focuses on four key strategies:
The role of Indian Ocean:  Enable transformative action by

RE
providing knowledge, resources, and
 As the Indian Ocean warms faster and
technical and institutional capacity to act
faster, it generates additional precipitation.
and supporting the sharing of information,
This draws more air from other parts of the
experience, and expertise.
world to the Indian Ocean, including the
Atlantic.  Mobilize support for action to put short-
lived climate pollutants on the policy map
 With so much precipitation in the Indian through advocacy at all levels of government
Ocean, there will be less precipitation in
CO
and in the private sector and civil society.
the Atlantic Ocean. Less precipitation will
lead to higher salinity in the waters of the  Increase the availability of and access
tropical portion of the Atlantic because there to financial resources to support the
won’t be as much rainwater to dilute it. successful implementation of scalable,
transformational action.
 This saltier water in the Atlantic, as it
comes north via AMOC, will get cold much  Enhance scientific knowledge to help
quicker than usual and sink faster. decision-makers scale up action and promote
the multiple benefits of action on short-lived
climate pollutants.
S

56. Correct Option: (d) Funding of CCAC:


Explanation:  The Coalition’s activities are financed
All statements are correct through a multi-donor The Climate and
GS


Clean Air Trust Fund, established in
Supplementary notes: 2012, which is administered through UN
Environment. While governments are the
Climate and Clean Air Coalition (CCAC): core of the Coalition’s funding, contributions
 The Climate and Clean Air Coalition is from the private sector and global community
a voluntary partnership of governments, are encouraged.
intergovernmental organizations, Goal of CCAC:
businesses, scientific institutions and
civil society organizations committed to  The Coalition’s goal is to reduce
improving air quality and protecting the short-lived climate pollutants beyond
climate through actions to reduce short- the recommendations made by the
lived climate pollutants. Intergovernmental Panel on Climate
Change (IPCC) in its special report Global
 Its global network currently includes over Warming of 1.5°C.
120 state and non-state partners, and
hundreds of local actors carrying out Benefits of increasing action on short-
activities across economic sectors. lived climate pollutants:
What are short-lived climate pollutants?  It can avoid an estimated 2.4 million
premature deaths from outdoor air pollution
 Short-lived climate pollutants are those annually by 2030/.
pollutants which are short lived in
atmosphere.  It can prevent as much as 52 million tonnes
of crop losses per year, and slow the increase
 They are also known as Super in global warming by as much as 0.6°C by
Pollutants. 2050.

PTS2020/PPP-16/112019/26 24
 It can also prevent the climate tipping points  Upadhyaya’s ideology for India:
that can exacerbate long-term climate
 Upadhyaya devised the political
impacts and make adapting to climate
change harder, especially for the poor and philosophy of Integral Humanism,
most vulnerable. which advocates the simultaneous and
integrated program of the body, mind
India and CCAC: and intellect and soul of each human
 India formally joined the Climate & being.
Clean Air Coalition (CCAC), becoming the  This philosophy of his is a synthesis
65th country to join the partnership. of the material and the spiritual; the
 India plans to work with Climate Clean and individual and the collective.
Coalition countries on best practices and  For India, he visualised a decentralised
experiences for the effective implementation
polity and self-reliant economy with
of India’s National Clean Air Programme
the village being the core basis which
(NCAP).
thought of India as an independent
nation which cannot rely upon western
57. Correct Option: (b) concepts like individualism, democracy,
socialism, communism or capitalism.
Explanation:

RE
 Deendayal was of the view that India is in
 Statement 3 is incorrect: He contested urgent need of a ‘fresh breeze’ to get rid of
for Lok Sabha from Uttar Pradesh, but did
the post-independence westernisation.
not get elected as he failed to gain enough
political traction.  He felt Indian intellect had been
suffocated, and Indian polity was no
Supplementary notes: more rooted in the traditions of our
 Deendayal’s personal life: O ancient culture.
 Deendayal Upadhyaya was born in a  Deendayal, just like all strong leaders,
village near the Farah town in Mathura believed in the concept of Swaraj
district called Chandrabhan, which (Self Governance).
later went on to be named after him as
“Deendayal Dham”.  Deendayal’s death: A mystery
SC
 His father’s name was Bhagwati Prasad,  A pool of doubt surrounds the mysterious
an astrologer, and his mother Rampyari death of this political leader.
was a religious woman.  Travelling in a train to Patna, a year
 Both his parents died when he was only after he was elected in 1967 as the
eight years old and he was then brought president of the Jana Sangh, he was
up by his maternal uncle and aunt who allegedly murdered.
took care of him and his education as
GS

 10 minutes after the train’s arrival at


well.
Mughal sarai station, his body was found
 Deendayal and the Bharatiya Jana near it clutching a five-rupee note in his
Sangh: hand.
 After meeting the RSS founder KB  Hence, the problematic case has been
Hedgewar in 1942, he joined RSS as a looked into by several judges and has
full-time worker, called Pracharak. seen demands made by several MPs,
 In 1951, Shyama Prasad Mukherjee politicians and family members to be
founded the Bharatiya Jana Sangh. reopened.
 Deendayal was seconded to the party by Deendayal Upadhyayas Literary work:
the RSS, bestowed with the responsibility
of moulding it into a genuine member of  Samrat Chandragupta (1946)
the ‘Sangh Parivar’.  Jagatguru Shankaracharya (1947)
 Then, he was appointed as General  Akhand Bharat Kyon? (1952)
Secretary of the party’s Uttar Pradesh
branch, and later as the all-India General  Bharatiya Arthniti: Vikas Ki Disha (1958)
Secretary.  The Two Plans: Promises, Performances,
 He remained the general secretary for 15 Prospects (1958)
long years and also contested for Lok  Rashtra Jivan Ki Samasyayen (1960)
Sabha from Uttar Pradesh, but did
not get elected as he failed to gain  Devaluation: A Great Fall (1966)
enough political traction.  Political Diary (1968)

25 PTS2020/PPP-16/112019/26
 Rashtra Chintan  The survey collects data on several
parameters like teachers, student
 Integral Humanism enrolment, programmes, examination
 Rashtra Jivan Ki Disha results, education finance,
infrastructure, etc.

58. Correct Option: (a)  The data collected under these parameters
come handy for making informed policy
Explanation: decisions and conducting research in
 Option (a) is correct: Crimea is situated education development.
between Black sea and Sea of Azov.  The survey is building a sound database,
Supplementary notes: which is getting updated annually as per
information submitted by the institutions
Crimea
Highlights of AISHE Report 2018-19
 The Crimean peninsula is connected on
the northwest to the mainland by the  Total enrolment in higher education has
“Perekop Isthmus”, a 5-mile- (8-km-) been estimated to be 37.4 million with
wide strip of land that has been the site of 19.2 million male and 18.2 million female.
numerous battles for the control of Crimea. Female constitute 48.6% of the total

RE
 Between Crimea and the mainland to enrolment.
the north lies Svyash (“Putrid Sea”), a  Gross Enrolment Ratio (GER) in
network of shallow inlets that is separated Higher education in India is 26.3%, which
from the Sea of Azov by the Arabat Spit, a is calculated for 18-23 years of age group.
70-mile- (113-km-) long sandbar along the GER for male population is 26.3% and
eastern shore of Crimea. for females, it is 26.4%. For Scheduled
 The Crimean Peninsula was annexed by Castes, it is 23% and for Scheduled
O
the Russian Federation in 2014 and since
then has been administered as two Russian
Tribes, it is 17.2% as compared to the
national GER of 26.3%.
federal subjects - the Republic of Crimea
and the federal city of Sevastopol.  UP and Karnataka have more female
enrolment in higher education and the
SC
ratio of male students is higher than females
on almost every level, except Certificate,
Post Graduate and M.Phil. The female
enrolment in Karnataka and UP is
50.04% and 50% respectively.
 Maharashtra has recorded the second-
highest student enrolment with roughly
54.95% male and 45.05% females.
GS

 In Rajasthan, male enrollment number


is higher as compared to the females.
 UG level student enrolment is 51%
males and 49% females across India.
 The top 6 States having highest student
enrollment are UP, Tamil Nadu,
Maharashtra, Rajasthan, Karnataka
and West Bengal. However, when it comes
59. Correct Option: (c)
to the total number of institutes, the state-
Explanation: wise ranking is different with UP (8077) at
the top followed by Maharashtra (6662),
 Statement 2 is incorrect: More male
Karnataka (5028), Rajasthan (3723),
teachers in Higher Educational Institutions
Andhra Pradesh (3540) and Tamil Nadu
(HEIs) in India than females. (3443).
Supplementary notes:  The above mentioned 6 states with
India’s Survey on Higher Education the highest student enrolment have
roughly 54.23% of the total student
 The survey, undertaken as an annual, enrolment in India
web-based, pan-India exercise on the
status of Higher Education since 2010-  Remaining 30 States (including UTs)
11, covers all the Higher Educational have only 25.76% of the total student
Institutions in the country. enrolment.

PTS2020/PPP-16/112019/26 26
 The survey covers all institutes of higher  The Bill sets up an Advisory Council
education in India, and the institutes are chaired by the Union Minister of
categorised in 3 broad categories such Human Resource Development. The
as College, University and Stand-alone Council will advise on coordination
institutes (offering Paramedical, Diploma, and determination of standards in
Hotel Management and Catering courses). higher education between the centre
and states.
 There are a total of 993 Universities,
39931 Colleges and 10725 Stand Alone Background
Institutions listed on the AISHE web portal
and out of that, 962 Universities, 38179  In India, education is included in the
Colleges and 9190 Stand Alone Institutions concurrent list which implies that both the
have responded during the survey. 298 centre and states can enact laws on it.
Universities are affiliating.  Higher education is regulated by multiple
authorities. The University Grants
 More male teachers in Higher
Commission (UGC) regulates universities
Educational Institutions (HEIs) in
and colleges teaching general subjects. It
India than females, with the lowest
has the power to determine and maintain
gender proportion in Bihar.
standards and disburse grants.
 4,16,299 is the total number of teachers

RE
 The All India Council for Technical
out of which about 57.8 per cent are male
Education (AICTE) regulates technical
teachers and 42.2 per cent are female
education such as engineering, management,
teachers. In Bihar, 78.97 per cent are
and architecture.
male teachers and 21.03 per cent are
females.  Further, institutions offering courses related
to medical, legal, nursing or teacher education
 Jharkhand has 69.8 per cent male teachers
are regulated by 14 professional councils such
and 30.2 per cent female teachers.


O
UP has 32.3 per cent, female teachers.
At All India level, there are just 73 female
teachers per 100 male teachers.

as the Medical Council, the Bar Council, and
the Nursing Council.
The several expert bodies such as the
National Knowledge Commission (2009),
the Yashpal Committee (2010), and the
SC
Committee for Evolution of the New
60. Correct Option: (d) Education Policy (2016) have suggested
measures to reform higher education to
Explanation:
address issues related to access, quality,
 All statements are correct funding and governance.
Supplementary notes:  Noting that the current system is
overregulated but under governed, they
Higher Education Commission
GS

recommended consolidating all existing


 The Draft bill on Higher Education regulators under an independent
Commission of India (Repeal of University regulator.
Grants Commission Act, 1956) Bill, 2018  In 2011, the Higher Education and
was made public to attract public opinion. Research Bill, 2011 was introduced in
 The Bill seeks to repeal the University Parliament which sought to consolidate
Grants Commission Act, 1956 and the regulators of all higher education under
establishes the Higher Education a single regulator. However, the Bill was
Commission of India (HECI). withdrawn in 2014.

 The HECI will maintain academic Key Issues and Analysis


standards in higher education by  HECI aims to promote autonomy of higher
specifying learning outcomes for courses, educational institutions. However,
specifying eligibility criteria for Vice certain provisions of the Bill do not meet
Chancellors, and ordering closure of higher this stated objective. It may be argued
educational institutions which fail to adhere that instead of granting higher educational
to minimum standards. institutions increased autonomy, the Bill
 Every higher educational institution provides HECI with extensive regulatory
empowered to award degrees or control.
diplomas will have to apply to the HECI  Currently, institutions offering professional
to commence its first academic operations. courses are regulated by 14 professional
The HECI also has the power to revoke councils. Of these, the Bill seeks to bring
permission on specified grounds. legal and architecture education within

27 PTS2020/PPP-16/112019/26
the purview of HECI. It is unclear why  The Golden Crescent is located in
only these two areas are included within the Southwest Asia and consists of three
regulatory ambit of the HECI and not the contiguous countries of Pakistan,
other fields of professional education. Afghanistan and Iran from East to
West.
 At present, the UGC has the power
to allocate and disburse grants to  Iran is the largest country, Afghanistan
universities and colleges. While the Bill the only land locked country and
replaces the UGC, it does not include any Pakistan shares the maximum land
provisions regarding disbursal of grants. boundaries in this region.
This raises a question whether HECI will
have any role in the disbursal of grants to  This is the world’s leading illicit opium
higher educational institutions. producing region with the potential
production of 5,020 metric tons, out of
 Presently, the Central Advisory Board which Afghanistan alone has 4,950 metric
of Higher Education (CABE) co- tons of illicit opium production, followed
ordinates and advises the centre by Pakistan with meager 70 metric tons in
and states on education related 2005 (UNODC, INCSR, 2006).
matters. The Bill creates an Advisory
Council and requires HECI to implement  Iran is not an opium producing country
its recommendations. This may restrict in this region but is more that of a

RE
HECI from functioning as an independent trans-shipment country.
regulator.

62. Correct Option: (c)


61. Correct Option: (a)
Explanation:
Explanation:
 Option (c) is correct: MAITREE-2019 was
 Option (a) is correct: Golden triangle held between India and Thailand.
O
comprises of Laos, Thailand and Myanmar.
Supplementary notes:
Supplementary notes:
MAITREE Exercise
Golden Triangle
 Exercise MAITREE is the annual bilateral
SC
 The Golden Triangle is located in Southeast joint exercise between Indian Army (IA) and
Asia comprising of three contiguous Royal Thailand Army (RTA).
countries of Laos, Thailand and Myanmar
from East to West.  The aim of exercise is joint training of
troops in counter terrorism operations in
both jungle terrain and urban scenario.
 Exercise MAITREE-2019 has been the face
of long standing bilateral ties between India
GS

and Thailand since 2006.


 The 14 days exercise will be conducted under
the framework of United Nations mandate.
Both contingents will share their valuable
experiences in the conduct of counter
terrorism operations in form of lectures,
drills, demonstrations and skill-at-arms
with an aim to refine tactics, techniques
and procedures.
 Myanmar is the largest country; Laos
is the only landlocked country not only
of the Golden Triangle but also of that of the 63. Correct Option: (a)
entire Southeast Asia and Thailand has
the largest coastline in the region. Explanation:
 The Golden Triangle is the world’s second  Statement 3 is incorrect: First Quad
largest illicit opium producing region talks were held in 2015 in Manila,
with a potential production of 5,020 metric Philippines.
tons out of which, Myanmar itself has
30,900 metric tons followed by 10,000 Supplementary notes:
metric tons in Laos and Thailand having the Quad Grouping
least production of 128 metric tons in 2005
(UNODC, INCSR, 2006). Laos is more that  Indo-Pacific Quadrilateral also known as
of trans-shipment country. Thailand and Quad Group is a closed group consisting of
Myanmar are money laundering countries. India, Australia, Japan & USA.

PTS2020/PPP-16/112019/26 28
 Japan pioneered the initiative in 2007 (200 to 1000 m). The distinction between
as a coalition of maritime democracies. the Upper Ocean and Deep Ocean depends
on the processes being considered.
 First Official quad talks were held in
Manila, Philippines in 2017  The ocean is a fundamental climate
regulator on seasonal to millennial time
 Securing a rules-based global order,
liberal trading system and freedom of scales. Seawater has a heat capacity
navigation are believed to be the guiding four times larger than air and holds
principles. vast quantities of dissolved carbon.

 It aims to contain a rising China and its  Heat, water, and biogeochemically relevant
“predatory” economic and trade policies. gases (e.g., oxygen (O2) and carbon dioxide
(CO2)) exchange at the air-sea interface,
 The purpose is also to provide alternative and ocean currents and mixing caused
debt financing for countries in the Indo- by winds, tides, wave dynamics, density
Pacific region. differences, and turbulence redistribute
 Report on stability was recently launched by these throughout the global ocean.
four think tanks from the Quad countries at
the Vivekananda International Foundation
65. Correct Option: (c)
(VIF).

RE
 The report was published by Quadripartite Explanation:
Commission on Indian Ocean Regional  Statements 1 is incorrect: Vitamin B12
Security, which put out a series of 20 policy is water soluble.
recommendations for stability in the Indian
Ocean Region (IOR). Supplementary notes:
 It recommended that the four countries Vitamin B12
should work to oppose the establishment
A water-soluble vitamin, B12 is the most
of permanent Chinese military bases in
the IOR.
O 
critical element in the metabolism of
every cell in the human body. Deficiency
can cause anemia, weakness of limbs and
64. Correct Option: (b) dementia.
SC
Explanation:  Pregnant women who lack this vitamin can
give birth to brain-damaged babies, which
 Statement 1 is incorrect: The ocean is can lead to autism.
vertically stratified with less dense
water sitting above more dense layers  India doesn’t have nation-wide data
on B12 deficiency. But there is wide
Supplementary notes: acceptance among the medical fraternity
that a large number of people as many as
About Global Oceans:
GS

those suffering from vitamin D deficiency


 The ocean contains about 97% of the have this problem.
Earth’s water, supplies 99% of the
Earth’s biologically-habitable space,  About 47 per cent people in northern India
and provides roughly half of the are vitamin B12 deficient. Thus, being an
primary production on Earth. endemic problem

 The average depth of the global ocean  This vitamin is only found in animal meat
is about 3700 m, with a maximum depth of and to a lesser extent in milk products.
more than 10,000 m.  For India, this debate has greater relevance
 The ocean is vertically stratified with as about 30 per cent of the population is
less dense water sitting above more vegetarian, according to an assessment of
dense layers, determined by the seawater the National Sample Survey Office in 2014.
temperature, salinity and pressure. The An Indian eats roughly 4 kg of meat per
surface of the ocean is in direct contact with year, as compared to the global average of
the atmosphere, except for sea ice covered 43 kg, according to the Food and Agriculture
regions. Organization.

 Sunlight penetrates the water column  But there is another layer to the curious
and supports primary production (by case of B12 deficiency in India. Even the
phytoplankton) down to 50 to 200 m depth non-vegetarian population is facing this
(epipelagic zone). deficiency. Most non-vegetarian Indians
largely consume chicken, which has very
 Atmospheric-driven mixing occurs from the little amount of vitamin B12 as compared to
sea surface and into the mesopelagic zone mutton or sea food

29 PTS2020/PPP-16/112019/26
 The government should also think about  Enhanced levels of international
fortification as a possible panacea. The competitiveness
easiest solution is to inject the vitamin; a
 Organization for increased production and
one course injection can suffice for a year.
productivity
 Achievement of a greater measure of
66. Correct Option: (d) economic leverage
Explanation:  Effectiveness of Member States in dealing
 All statements are correct with Third States, groups of States and
entities of any description
Supplementary notes:
 The enhanced coordination of Member
Caribbean Community and Common States’ foreign and foreign economic policies
Market (CARICOM) and enhanced functional cooperation.
 The Caribbean Community and Common
Market (CARICOM) is a group of twenty 67. Correct Option: (b)
developing countries in the Caribbean
that have come together to form an economic Explanation:
and political community that works Statement 1 is incorrect: Dehradun

RE

together to shape policies for the region and based Wildlife Institute of India (WII)
encourages economic growth and trade. has launched a mobile application “Ganga
Seat of Secretariat: Georgetown, Guyana Data Collector” under the “Biodiversity
and Ganga Conservation” project initiated
 Fifteen of these countries are full-fledged by the National Mission for Clean Ganga
members of the community while five of (NMCG) of Union Ministry of Jal Shakti.
them only retain associate member status.
Supplementary notes:

O
The fifteen full-time countries are Antigua
and Barbuda, Bahamas, Barbados, Belize, Ganga data collector app
Dominica, Grenada, Guyana, Haiti, Jamaica,
 The app will provide field researchers with
Montserrat, Saint Lucia, Saint Kits and
a complete data entry solution to monitor
SC
Nevis, Saint Vincent and the Grenadines,
the aquatic population in the river.
Suriname and Trinidad and Tobago. The
associate members are Anguilla, Bermuda,  Dehradun based Wildlife Institute
British Virgin Islands, Cayman Islands and of India (WII) has launched a mobile
Turks and Caicos. application “Ganga Data Collector” under
the “Biodiversity and Ganga Conservation”
 These nations have collectively joined
together to expand their trade and economic project initiated by the National Mission for
relations internationally, including further Clean Ganga (NMCG) of Union Ministry of
Jal Shakti.
GS

development of activity in international


markets.  The Ganga basin covers 11 states
 CARICOM was formed in 1973 after including Uttarakhand, Uttar Pradesh,
the founders had enacted the Treaty of Madhya Pradesh, Rajasthan, Haryana,
Chaguaramas. Himachal Pradesh, Chhattisgarh,
Jharkhand, Bihar, West Bengal and
 The Caribbean Community and Delhi.
Common Market was established to
replace the Caribbean Free Trade  In the first phase, training will be given to
Area which had failed in its mission volunteers and staff in five Ganga states
to develop policies in the region including Uttarakhand, Uttar Pradesh,
pertaining to labor and capital. Bihar, Jharkhand and West Bengal where
Ganga stretches over 2,200-km. In the
Pillars of Integration second phase, training will be given to the
 To improve standards of living and staff of the other six states of the Ganga
work basin.

 Full employment of labor and other factors


of production 68. Correct Option: (d)
 Accelerated, coordinated and sustained Explanation:
economic development and convergence
 Statement 1 is incorrect: It is a polar
 Expansion of trade and economic relations expedition initiated by German research
with Third States icebreaker Polarstern.

PTS2020/PPP-16/112019/26 30
 Statement 2 is incorrect: It is a one-year-  Plants like lemongrass, agave, rambans,
long expedition into the Central Arctic to and certain species of chilly including some
study its climate. other plant rare species have been identified
as effective to keep wild animals away.
Supplementary notes:
 The government is planning to grow these
Mosaic Mission plants in the areas where wild animals
enter residential areas near forests.
 The Multidisciplinary drifting Observatory
for the Study of Arctic Climate expedition  Wild animals like leopards and bears, along
is a one-year-long expedition into the with elephants and wild boars are a major
Central Arctic, planned to take place from threat to human life, livestock and crops in
2019 to 2020. such areas the forest officials informed.

 For the first time a modern research


icebreaker will operate in the direct vicinity 70. Correct Option: (a)
of the North Pole year round, including the
Explanation:
nearly half year long polar night during
winter.  Option (a) is correct: Sagittarius A*
is one of very few black holes in the
 During its one-year-long journey, the universe where we can actually witness the
central expedition ship, the research

RE
flow of matter nearby.
icebreaker Polarstern from Germany’s
Alfred Wegener Institute, Helmholtz Supplementary notes:
Centre for Polar and Marine Research
Sagittarius A
(AWI), will be supported and resupplied
by the icebreakers Akademik Fedorov and  Sagittarius A is a bright and very compact
Admiral Makarov (Russia), Oden (Sweden) astronomical radio source at the center
and Xue Long II (China). O of the Milky Way, near the border of the
constellations Sagittarius and Scorpius.
 The `results of MOSAiC mission will
contribute to enhance understanding of the  It is likely the location of a supermassive
regional and global consequences of Arctic black hole, similar to those generally
accepted to be at the centers of most if not
climate change.
all spiral and elliptical galaxies.
SC
 It will be helpful in understanding the
 Astronomers have used NASA’s Chandra
reasons behind the sea-ice loss and improve
X-ray Observatory to take a major step in
weather and climate predictions. understanding why material around Sgr A*
is extraordinarily faint in X-rays.
69. Correct Option: (c)  Sgr A* is one of very few black holes in the
universe where we can actually witness the
Explanation: flow of matter nearby.
GS

 Both statements are correct

Supplementary notes: 71. Correct Option: (a)


Bio-fences Explanation:
 Bio-fences/Live fences are lines of trees  Pair 2 is incorrectly matched: Tilaiya
or shrubs planted on farm or field dam was constructed across the Barakar
boundaries that provide protection River at Tilaiya in Koderma district in
against cattle and wildlife, act as Jharkhand. It was the first of the four multi-
purpose dams included in the first phase of
windbreaks, enrich the soil, provide
the Damodar Valley Corporation.
bee forage, provide shade, and control
dust. Hence it is environment friendly  Pair 3 is incorrectly matched: Bansagar
and economical for farmers. Dam is a multipurpose river Valley Project
on Sone River situated in the Ganges Basin
 They are less expensive and more useful in the state of Madhya Pradesh.
than fences made of wood, barbed wire, or
stone masonry. Various species have been Supplementary notes:
tested to discover their suitability for use as
Khadakwasla Dam
biofencing plants.
 Khadakwasla Dam was built on the Mutha
 The Uttarakhand government is planning River. The dam created a reservoir known
to us this unique and cost-effective method as Khadakwasla Lake which is the main
to keep wild animals away from residential source of water for Pune and its suburbs in
areas. the state of Maharashtra.

31 PTS2020/PPP-16/112019/26
 Tilaiya dam was constructed across the Hospitals wherein the influx of patients is
Barakar River at Tilaiya in Koderma district more.
in Jharkhand. It was the first of the four
 To produce skilled clinicians in Human
multi-purpose dams included in the first
Genetics.
phase of the Damodar Valley Corporation.
 To undertake screening of pregnant women
 Bansagar Dam is a multipurpose river
and new born babies for inherited genetic
Valley Project on Sone River situated in
diseases in hospitals at aspirational
the Ganges Basin in the state of Madhya
districts.
Pradesh.
Why need UMMID?
 Other major Dams and Reservoirs in India:
 The contribution of genetic disorders
to infant and childhood mortality and
morbidity is increasing.
 In India’s urban areas, congenital
malformations and genetic disorders are
the third most common cause of mortality
in new-borns. With a very large population
and high birth rate, and consanguineous

RE
marriage favoured in many communities,
prevalence of genetic disorders is high in
India.
 The congenital and hereditary genetic
diseases are becoming a significant health
burden in India, and there was the need for
adequate and effective genetic testing and
O counselling services.
 Though there are excellent medical genetics
centres in India providing state-of-the-art
patient care services to families with genetic
SC
disorders, the number of such centres is
small.
 Hence establishment of patient care
services for genetic disorders is the need of
the time.
 Therefore the Department of Biotechnology
has initiated the UMMID program to
72. Correct Option: (c) establish many genetic diagnostic centres
GS

in different parts of the country.


Explanation:
 To achieve this, a combined program of
 Both statements are correct
training doctors in genetic diagnostics
Supplementary notes: and providing funding to establish
genetic diagnostic laboratories and to
UMMID create awareness about genetic disorders
 The Department of Biotechnology (DBT) amongst doctors and lay persons, has been
has started the UMMID (Unique Methods meticulously planned.
of Management and treatment of
Inherited Disorders) Initiative which is 73. Correct Option: (a)
designed on the concept of ‘Prevention is
better than Cure’. Explanation:
 UMMID aims to create awareness about  Statement 2 is incorrect: It was during
genetic disorders amongst clinicians to the reign of King Narasimha Varman I
establish molecular diagnostic in hospitals. (630 - 668AD), the name Mahabalipuram
was changed to Mamallapuram.
Goals of UMMID initiative:
Supplementary notes:
 To establish NIDAN (National Inherited
Diseases Administration) Kendras Mamallapuram
to provide counselling, prenatal testing
and diagnosis, management, and  Mahabalipuram was the second capital
multidisciplinary care in Government of the Pallavas who ruled Kanchipuram.

PTS2020/PPP-16/112019/26 32
Formerly, mahabalipuram was known and  The first Tejas train was started
called as Mamallapuram. between Lucknow to Delhi.
 King Narasimha Varman I was a great  Antyodaya Express is a completely
and valiant warrior. He was given unreserved train designed by Indian
the title Mamalla which means Railways. The word ‘Antyodaya’ refers
‘the great wrestler’ so the name to uplifting of the weakest section of the
was converted from Mahabalipuram society. These are supposed to be overnight
to Mamallapuram considering the great fully unreserved trains.
king and his achievements.
 Indian Railways introduced the first
 During the reign of the Pallava dynasty, Antyodaya Express between Ernakulam
between the 3rd century CE and 7th century and Howrah.
CE, it became an important centre of art,
architecture and literature.  UDAY Express or Utkrisht Double
Decker Air Conditioned Yatri Express
 Mahabalipuram was already a thriving sea are completely Double-Decker AC chair car
port on the Bay of Bengal before this time. trains designed by Indian Railways.
A significant amount of coins and other
artefacts excavated from this region also  The first UDAY Express train started
indicate a pre-existing trade relation with service between Coimbatore and Bangalore
in the month of June 2018

RE
the Romans even before it became a part of
the Pallava Empire.  These all categories of trains were
announced in the budget 2016-2017.
Art & Architectural Masterpieces at
Mamallapuram:
 Shore Temple: The Shore Temple (built 75. Correct Option: (c)
in 700–728 AD) is so named because it Explanation:
overlooks the shore of the Bay of Bengal.
O
It is a structural temple, built with blocks
of granite, dating from the 8th century
AD. It was built during the reign of
Narasimhavarman II. It is a remnant
 Both statements are correct
Supplementary notes:
PMKSY
of a larger complex of temples and civil
SC
structures much of which lie under the  A Central Sector Scheme - SAMPADA
depth of the sea now. (Scheme for Agro-Marine Processing and
Development of Agro-Processing Clusters)
 Descent of the Ganges: It is alternatively was approved by the Cabinet Committee on
known as Arjuna’s Penance. Descent of Economic Affairs (CCEA) in 2017.
the Ganges is a gigantic open air bas-relief
sculpted out of pink granite. The dramatic  This umbrella scheme has now been
relief sculpture narrates the tales from renamed as the “Pradhan Mantri
Kisan Sampada Yojana (PMKSY)” to
GS

Indian epics such as the Mahabharata.


be implemented by Ministry of Food
 Pancha Ratha: Pancha Ratha (five chariots) Processing Industries (MoFPI).
is an architectural ode to Mahabharata’s
five Pandava brothers Yudhistir, Bhima,  PM Kisan SAMPADA Yojana is a
Arjuna, Nakula, Sahadeva, and their wife comprehensive package which aims
Draupadi. Thematically and structurally, to create modern infrastructure with
each ratha is significantly different from efficient supply chain management
the other ones, but all of them were carved from farm gate to retail outlet.
out of a long stone or monolith  It will not only provide a big boost to the
growth of food processing sector in the
74. Correct Option: (d) country but also help in providing better
returns to farmers and is a big step towards
Explanation: doubling of farmers income, creating huge
employment opportunities especially in the
 All statements are correct
rural areas, reducing wastage of agricultural
Supplementary notes: produce, increasing the processing level
and enhancing the export of the processed
Tejas Express foods.
 The Tejas Express is India’s first semi- Objectives of PMKSY:
high speed fully air-conditioned train
Introduced by Indian Railways. It features  Creation of modern infrastructure for food
modern on-board facilities with automatic processing mega food parks/ clusters and
doors. individual units.

33 PTS2020/PPP-16/112019/26
 To create effective backward and forward Supplementary notes:
linkages - linking farmers, processors and
markets. Ratapani Tiger Reserve

 To create robust supply chain infrastructure  Recently it is declared the Tiger State of
for perishables. India, Madhya Pradesh is inching closer
to getting its seventh tiger reserve — the
The following schemes will be implemented Ratapani Tiger Reserve.
under PM Kisan SAMPADA Yojana:
 Established in the year 1976, the Ratapani
 Mega Food Parks Wildlife Sanctuary boasts a forest area of
 Integrated Cold Chain and Value Addition around 688 sq km – mainly comprising a
Infrastructure beautiful teak forest.

 Creation/ Expansion of Food Processing/  The Ratapani Tiger Reserve, located in


Preservation Capacities (Unit Scheme) the Raisen district of Madhya Pradesh, in
Vindhya Range.
 Infrastructure for Agro-processing Clusters
 Other tree species found here include
 Creation of Backward and Forward bijasal, saj, tendu and khair.
Linkages
 Although the tiger holds the position of
 Food Safety and Quality Assurance top predator in the park, leopards, striped

RE
Infrastructure hyenas, jackals, Indian foxes and wild dogs
 Human Resources and Institutions also inhabit its forested tracts.
 Members of the deer family such as
76. Correct Option: (a) chausingha, chinkara, and barasingha are
spotted often, and you can also expect to see
Explanation: herds of nilgai, the largest Indian antelope.
 Option (a) is correct
O  Around 150 species of birds can be
spotted here. Birders can expect to spot
Supplementary notes: the Indian robin, white spotted fantail
About Kaniyan Pungundranar: fly-catcher, crimson breasted barbet
and the State Bird of MP – paradise
SC
 Kaniyan Poongunranar, also Poongundranar flycatcher, amongst other species.
or Pungundranar was an influential Tamil
philosopher from the Sangam age.  Many rare migratory birds from Tibet,
Nepal, Russia and Siberia also visit this
 His name Kaniyan implies that he was an sanctuary and are a great attraction.
astronomer as it is a Tamil word referring
mathematics.  Dahod Dam, Ratapani Dam and Barna Dam
are large water bodies inside the park and
 He was born and brought up in home to aquatic avifauna such as ducks,
GS

Mahibalanpatti, a village panchayat in the geese, cranes, storks and snipes. Among
Thiruppatur taluk of Sivaganga district in reptiles, pythons and crocodiles are also
the state of Tamil Nadu. seen here.
 He composed two poems in Purananuru
and Natrinai.
78. Correct Option: (c)
 His famous Tamil quote “Yaadhum Oore
Yaavarum Kelir” meaning “To us all Explanation:
towns are one, all men our kin” is at  Both statements are correct
present depicted in the United Nations
Organisation and also quoted by Prime Supplementary notes:
Minister Narendra Modi in United Nations
Goldschmidt
General Assembly address.
 A new, curious mineral has been discovered
 Abdul Kalam was the first one to quote this
inside a diamond unearthed from a mine in
famous quote in European Union Historical
South Africa.
Speech.
 The mineral has been named goldschmidtite,
after Victor Moritz Goldschmidt, the
77. Correct Option: (c) Norwegian scientist acknowledged as the
Explanation: founder of modern geochemistry.

 Statement 1 is incorrect: The Ratapani  Goldschmidtite has an unusual chemical


Tiger Reserve, located in the Raisen district signature for a mineral from Earth’s
of Madhya Pradesh, in Vindhya Range. mantle

PTS2020/PPP-16/112019/26 34
 While the mantle is dominated by elements Supplementary notes:
such as magnesium and iron, goldschmidtite
has high concentrations of niobium, EEHV
potassium and the rare earth elements  Elephant endotheliotropic herpesviruses
lanthanum and cerium. (EEHV) or Elephantid betaherpesvirus 1
(ElHV-1) is a type of herpesvirus, which
can cause a highly fatal hemorrhagic
79. Correct Option: (b) disease when transmitted to young
Explanation: Asian elephants.

 Statement 1 is incorrect: Bathukamma  The four deaths in Nandan Kanan Zoo are
is a colourful and vibrant festival of the first reported cases of EEHV-related
Telangana and celebrated by women, with deaths in an Indian zoo
flowers that grow exclusively in each region.  Most elephants carry just as most humans
This festival is a symbol of Telangana’s carry a cold virus. When EEHV is
cultural identity. triggered, the elephant dies of massive
internal bleeding and symptoms which
Supplementary notes: are hardly visible.
Bathukamma  Some elephants show symptoms such as
reduced appetite, nasal discharge and

RE
 Bathukamma is a colourful and vibrant
festival of Telangana and celebrated by swollen glands
women, with flowers that grow exclusively  There is no true cure for herpesviruses
in each region. This festival is a symbol of in animals or in humans, because
Telangana’s cultural identity. herpes viruses go latent.
 The festival begins a week before the grand  EEHV is lethal for young elephants between
‘Saddula Batukamma’ (the grand finale of the ages of one and 12. If a young elephant
the Batukamma festival) which falls two


days before Dussehra.
O
Bathukamma is a beautiful flower stack,
arranged with different unique seasonal 
dies before reproducing, it affects the
population of the species as a whole in the
concerned geography.
In India, the CZA will set up a national
flowers most of them with medicinal values, committee of scientists from Guwahati,
SC
in seven concentric layers in the shape of Kerala, IVRI and Nandan Kanan to develop
temple gopuram. protocols for the country lest an EEHV
outbreak occurs elsewhere in the future.
 Batukamma celebrates the inherent
relationship between earth, water and the
human beings. 82. Correct Option: (a)
Explanation:
80. Correct Option: (a)
GS

 Option (a) is correct: Bru refugees


Explanation: belong to Mizoram
 Option (a) is correct Supplementary notes:
Supplementary notes: Bru refugees
INS Nilgiri  Mizoram officials have re-identified 26,128
Bru refugees belonging to 4,278 families
 INS Nilgiri is the first ship of Project 17A lodged in six relief camps in North Tripura
frigates district
 Project 17A frigates are a design derivative  The Brus are in Tripura since late 1997 in
of the Shivalik class stealth frigates the wake of a communal tension triggered
with much more advanced stealth features by the murder of a forest guard inside the
and indigenous weapons and sensors. Dampa Tiger Reserve on October 21, 1997 by
 The P17A frigates incorporate new design Bru National Liberation Front militants.
concepts for improved survivability,  The Brus, also referred to as the Reangs,
sea keeping, and stealth and ship are spread across the northeastern states of
manoeuvrability. Tripura, Assam, Manipur, and Mizoram.

81. Correct Option: (d) 83. Correct Option (a)


Explanation: Explanation:
 All statements are correct  Option (a) is correct:

35 PTS2020/PPP-16/112019/26
Supplementary notes:  The organization holds an annual
international conference and several
Section 11 of RPA, 1951 and related news regional conferences in different parts
 Sikkim Chief Minister Prem Singh of the world.
Tamang was barred by law from  World Hindu Economic Forum has earlier
contesting elections for six years in been organised in London, Chicago, Losa
2016 for graft charges.
Angeles, Hong Kong, and Nairobi.
 He had been sentenced to one year
 2019 World Hindu Economic Forum
imprisonment for misappropriation of
held from 27-29 September in Mumbai.
funds in the procurement of cows when
he was state Animal Husbandry Minister  Dharmasya Moolam Arthah (Economy
under the Prevention of Corruption Act, is the strength) is the philosophy of
1988. However, the section under which WHEF. (Kautilian Arthashastra)
he was convicted had been omitted in an
amendment in 2018. He completed the one-
year sentence on August 10, 2018. 85. Correct Option: (a)
 After completing his one-year prison term Explanation:
in 2018 his disqualification was sharply
 Option (a) is correct

RE
reduced to just a year and a month by
the Election Commission of India (ECI) Supplementary notes:
recently.
Special Report on Ocean and Cryosphere
 EC cited Section 11 of the Representation in a Changing Climate
of the People Act, 1951 which empowers
it to remove/reduce disqualification  The IPCC approved and accepted this report
prescribed by statutory rule for reasons at its 51st session held in September.
to be recorded.
O  Created in 1988 by the World Meteorological
 The EC justified its decision on the ground Organization (WMO) and the United
that Tamang was convicted of an Nations Environment Programme (UNEP)
offence committed in 1996-97, when  Headquarter: Geneva
a minimum punishment of two years
SC
was warranted under the Prevention  Objective: To provide governments
of Corruption Act, 1988, for attracting at all levels with scientific information
the six-year disqualification. Tamang was that they can use to develop climate
imprisoned for a year. The EC also noted policies. IPCC reports are also a key
that he had already served his sentence, and input into international climate change
hence, his case merited a disqualification negotiations.
waiver.
IPCC Reports
GS

84. Correct Option: (c) Reports Year


First 1990
Explanation:
Second 1995
 Statement 2 is incorrect: WHEF is a non-
profit organization headquartered in Delhi, Third 2001
India. Fourth 2007
Supplementary notes: Fifth 2013

World Hindu Economic Forum


 WHEF is a non-profit organization
86. Correct Option: (a)
headquartered in Delhi, India. Explanation:
 It was founded by Swami Vigyananand  Option (a) is correct.
in 2010.
Supplementary notes:
 It is as an independent international
organization committed to make Hindu About Extinction Rebellion
society prosperous through creation
and sharing of surplus wealth.  Extinction Rebellion is an international
movement that uses non-violent civil
 The forum brings together eminent Hindu disobedience in an attempt to halt
intellectuals and businesses for the mass extinction and minimise the risk
purposes of collaboration. of social collapse.

PTS2020/PPP-16/112019/26 36
 It is a movement made up of people from all  Farmers usually store their produce in
walks of life. It started in response to the moisture-proof and dust-proof structures
IPCC report that we only have 12 years to to prevent the bulbs from sprouting green
stop catastrophic climate change and our shoots.
understanding that we have entered the 6th
 Depending on the price, farmers release
mass extinction event.
their produce, enabling steady supply for
 It started small and was launched in the retail markets.
UK on October 31st 2018, it has grown
Why the price is rising?
very quickly and there are now about 130
Extinction Rebellion groups across the UK  Hoarding: Hoarding of perishable items is
and across the world. illegal; however, many traders have hoarded
onions and are now selling at much inflated
 The majority of people who work for
prices than before. Therefore, traders and
Extinction Rebellion are volunteers; a small
not farmers are benefited when onion prices
percentage is receiving living expenses.
hike.
 Decisions are made based on a model of
 Low Onion Consumption: The price
organising called a “self-organising system”
rise comes during months when onion
in which people do what they can accord the
consumption usually goes down. It goes with
skills and time they can offer.
the law of demand and supply. For example,

RE
it happens during the nine days of Navratri
87. Correct Option: (d) in Maharashtra and the month of Shravan.

Explanation:  Delayed Monsoon leads to supply-side


constraints: The current increase in onion
 All statements are correct prices is a fall out of 2018’s drought and
the delayed monsoon in 2019. Some
Supplementary notes: O onion-growing areas have reported excessive
Onion Pricing rain and delay in harvest period by a week
or so.
 Onion price has been shooting in the
wholesale and retail markets and the  A combination of factors has led to the
increase in prices now.
SC
central government has taken many efforts
to control the increasing price.  The cultivation area under rabi crop
 Since May 2019, prices in wholesale has decreased in Maharashtra. So,
markets across the onion-growing it reported a low cultivation in the rabi
districts of Maharashtra have been season of 2018-19.
increasing.  Karnataka received heavy rain
 The ripple effect can be seen in the retail during kharif crop’s harvest period,
markets where the price of onion has also which delayed the arrival of kharif
GS

increased. onions from Karnataka.

 According to Price Monitoring Cell (PMC) of  So, the stored rabi crop should have to be
the Consumer Affairs department, the retail supplied to the markets.
price of onion has increased by Rs 20-25/kilo  As the imported onions may not
across the country in the last six months (as arrive soon, the price of the bulb is
of September 2019). expected to stay high.
 Maharashtra, Karnataka, Gujarat, Madhya Steps Taken by Government to control
Pradesh, Rajasthan, Bihar, Uttar Pradesh Onion Prices
and West Bengal account for almost 90% of
onion production in the country.  In early September 2019, the state-
run Minerals and Metals Trading
 Maharashtra contributes nearly one Corporation of India (MMTC) had
third of the country’s production. floated contracts for importing onions
 Farmers across the country take three from Pakistan, Egypt, China, Afghanistan,
crops of onions which ensure uninterrupted etc. Following sharp criticism, MMTC
supply across the year. Of these, dropped Pakistan from the list of countries.

 Kharif and late kharif are not  The Centre also tried to restrict exports
amenable for storage (has higher by sharply hiking the Minimum Export
moisture content) Price (MEP).

 Rabi crop can be stored for a long  In June 2019, the government ended the
period. 10% export subsidy for onion.

37 PTS2020/PPP-16/112019/26
 Anticipating a shortage, the central  Before he died in 1539, he appointed a
government had created a buffer stock of follower called Lena (known later as Guru
57,000 tonnes, of which some have already Angad) as his successor.
been offloaded.
 Guru Angad compiled the work of Guru
Nanak and added to it his own script known
88. Correct Option: (c) as Gurmukhi.

Explanation:  The next three Gurus also wrote under


Nanak’s name, and their work was compiled
 Statement 3 is incorrect: Rafale UAV is by Guru Arjan in 1604.
not an anti-drone technology.
 Works of Shaikh Farid, Sant Kabir, Bhagat
Supplementary notes: Namdev, and Guru Tegh Bahadur were also
Various Anti-Drone Technologies added to these compilations.

 Drone Gun: A drone gun is capable of  Finally, in 1706, Guru Gobind Singh
jamming the radio, global positioning authenticated the compilation and
system (GPS) and mobile signal between created the Guru Granth Sahib.
the drone and the pilot and forces the drone  Guru Nanak’s followers increased in the
to ground in good time before it could wreak sixteenth century as people from all castes

RE
any damage and occupations joined the community.
 Sky Fence: It uses a range of signal  By the seventeenth century, the town of
disruptors to jam the flight path and Ramdaspur developed around a Gurudwara
prevent them from entering their target, a called the Harmandar Sahib and became a
sensitive installation or event venue. self-governing society.
 ATHENA: It is an acronym for Advanced  The Mughal Emperor Jehangir saw them
Test High Energy Asset, is another
weapon under analysis as it works by
O as a potential threat, and in 1606 he
ordered the execution of Guru Arjan.
firing a high energy laser beam on a
rogue drone resulting in its complete  By the seventeenth century, Guru Gobind
destruction in the air. However, this is Singh organised the Khalsa Movement,
SC
a very costly technology and is being and the community of Sikhs became a
currently tested by the US army. political organisation called the Khalsa
Panth.
 Drone Catcher: It swiftly approaches an
enemy drone and grabs it by throwing Guru Nanak’s Teachings:
a net around. Such a tool is required when
a rogue drone is needed to be captured safely  Guru Nanak emphasised the worship of
to extract incriminating evidence from it. one God and insisted that caste, creed,
and gender were irrelevant to attaining
Skywall 100: It is the ground version
GS

 salvation.
of the ‘drone catcher’ and it works by
bringing down an UAV using a parachute  According to him, pursuit of an active life
that is hurled through a net from 100 meters would liberate humans, and he also used the
distance. terms nam, dan, and isnan to mean right
worship, welfare, and purity of conduct.

89. Correct Option: (d)  His ideas today are called nam-japna, kirt-
karna, and vand-chhakna, which mean
Explanation: right belief and worship, honest living, and
 All statements are correct helping others.
 Thus, Guru Nanak’s idea of equality had
Supplementary notes:
social and political implications.
Guru Nanak
 Guru Nanak was born in Talwandi but he 90. Correct Option: (b)
travelled for years before he founded the
Dera Baba Nanak in Kartarpur. Explanation:
 Irrespective of their creed or caste, his  Statement 1 is incorrect: National
followers ate together in a common nutrition survey was conducted by the
kitchen known as langar. And the sacred Ministry of Health and Family Welfare in
place/temple he created for worship and partnership with United Nations Children
meditation was called dharmasal (now Fund (UNICEF) to measure the level of
known as Gurudwara). malnutrition in India.

PTS2020/PPP-16/112019/26 38
Supplementary notes: Supplementary notes:
Government Scheme Global Competitiveness Report
 National nutrition survey was conducted  The Global Competitiveness Report (GCR)
by the Ministry of Health and Family is a yearly report published by the World
Welfare in partnership with United Nations Economic Forum.
Children Fund (UNICEF) to measure the  The report “assesses the ability of countries
level of malnutrition in India. to provide high levels of prosperity to
 CDSA – Clinical Development Services their citizens”. This in turn depends on
Agency was selected as the monitoring how productively a country uses available
agency for the aforementioned survey. resources.

 The survey covered more than 1,20,000  The report measures twelve pillars of
covering both children and adolescents competitiveness. These are:
in rural and urban area. The survey was  Institutions
conducted over a period of two years (2016-
2018)  Appropriate infrastructure

 It aimed to assess the following  Stable macroeconomic framework


Good health and primary education

RE
 Micro nutrient deficiencies 
 Higher education and training
 Sub clinical inflammation
 Efficient goods markets
 Overweight or obesity
 Efficient labor markets
 Cardio – metabolic risks
 Developed financial markets

91. Correct Option: (c) Ability to harness existing technology

Explanation:
 Both statements are correct
O 
 Market size—both
international
domestic and

 Production of new and different goods


SC
Supplementary notes: using the most sophisticated production
processes
FAME 2 Scheme
 Innovation
 The outlay of Rs. 10,000 crore has been
made for three years till 2022 for FAME 2
scheme. 93. Correct Option: (d)
 The government will offer the incentives Explanation:
for electric buses, three-wheelers and four-
GS

 Statement 1 is incorrect: Lithium-ion


wheelers to be used for commercial purposes.
battery uses petroleum coke layered with
Plug-in hybrid vehicles and those with a
lithium ions and not lead-acid. Lead-acid
sizeable lithium-ion battery and electric batteries are still used to start gasoline- and
motor will also be included in the scheme diesel-powered vehicles today.
and fiscal support offered depending on the
size of the battery. Supplementary notes:
 The centre will invest in setting up charging Lithium-ion Battery
stations, with the active participation of
 The first rechargeable battery came about
public sector units and private players.
in 1859. These were made from lead-acid,
 FAME 2 will offer incentives to and are still used to start gasoline- and
manufacturers, who invest in developing diesel-powered vehicles today.
electric vehicles and its components,
 Dr. Stanley Whittingham discovered an
including lithium-ion batteries and electric
extremely energy-rich material, which he
motors. used to create an innovative cathode in a
lithium battery. This battery was made
92. Correct Option: (d) from titanium disulphide.
 The battery’s anode was partially made
Explanation:
from metallic lithium, which has a strong
 Option (d) is correct: Global drive to release electrons. It resulted in a
Competitiveness Index (GCI) is published battery that had great potential, just over
by World economic forum. two volts.

39 PTS2020/PPP-16/112019/26
 The big advantage of this technology was  When the battery is being recharged, lithium
that lithium-ion stored about 10 times as ions flow in the opposite direction, resetting
much energy as lead-acid or 5 times as the battery to do it all over again.
much as nickel-cadmium
Issues:
 Lithium-ion batteries were also  More than half of lithium is gathered using
extremely lightweight and required little brine extraction from deep inside the
maintenance. earth, and the rest is still mined traditionally
 Lithium ion batteries using cobalt oxide can from rock.
boost the lithium battery’s potential to four  Lithium extraction has caused
volts. environmental damage to areas around
lithium processing operations.
Benefits and uses of Lithium ion battery:
 The advantage of lithium-ion batteries
is that they are not based upon chemical
94. Correct Option: (c)
reactions that break down the electrodes, Explanation:
but upon lithium ions flowing back and
 Statement 3 is incorrect: All girl children
forth between the anode and cathode.
within the age of 13 to 18 in the state are
 They are lightweight, rechargeable, targeted beneficiaries.

RE
powerful batteries, now used in
everything from mobile phones
Supplementary notes:
to laptops and long-range electric Kanyashree Scheme
vehicles.
 Kanyashree is a conditional cash
 Battery technology helps replace carbon- transfer scheme aiming at improving
emitting sources because it allows power the status and wellbeing of the girl child
companies to store excess solar and wind by incentivising schooling of teenage
O
power when the sun does not shine nor the girls and delaying their marriages
wind blow, making possible a fossil fuel-free until the age of 18.
society and combating the effects of climate  It received the United Nations Public
change. Service Award last year.
SC
 They are also capable of being miniaturized  It Improve lives and status of the adolescent
and used in devices like implanted girls in State. Provide financial help to girls
pacemakers. from disadvantaged families to pursue
higher studies (now it will cover every girl
 They can be scaled up to power a car or a from state).
home.
 Prevent child marriage. Improved outcomes
 This battery technology helps replace in terms of their health (especially facilitate
carbon-emitting sources because it allows the prevention of infant and maternal
GS

power companies to store excess solar mortality).


and wind power when the sun does not
 Contribute towards empowerment of girls
shine nor the wind blow, making possible a
in the state. Bring immeasurable benefits
fossil fuel-free society and combating the
for the larger society as a whole.
effects of climate change.
 Components of Scheme Annual
Mechanics of Lithium ion battery: Scholarship of Rs. 500: It is for unmarried
girls aged 13-18 years enrolled in classes
VIII-XII in government recognized
regular or equivalent open school or
equivalent vocational or technical
training course.
 One time Grant of Rs. 25,000: It is for girls
turned 18 at time of application, enrolled
in government recognized regular or open
school and colleges or pursuing vocational
or technical training or sports activity or is
 Lithium-ion batteries are powered by flows inmate of Home registered under JJ Act.
of lithium ions crossing from one material
to another.  Progress: So far around 50 lakh girls from
the state have been benefited by scheme
 When the battery is in use, positively- and removal of ceiling will help another
charged lithium ions pass from an anode to three lakh more girls annually. Since its
a cathode, releasing a stream of electrons launch, it has helped to reduce number of
along the way that form an electric current. school drop out of girls by nearly 11.5%

PTS2020/PPP-16/112019/26 40
95. Correct Option: (b) attached to existing ones meant for solid
and wet waste.
Explanation:
 A three-month pilot project, the clinic, if
 Statement 2 is incorrect: It is listed successful, would be replicated elsewhere in
as ‘near threatened’ in the Red List of the country.
International Union for Conservation of
Nature (IUCN).
97. Correct Option: (b)
Supplementary notes:
Explanation:
Emperor Penguins
 Statement 1 is incorrect: Rajasthan has
 Emperor penguins are the tallest and announced the creation of a Pneumoconiosis
heaviest of all living penguin species and Fund.
are endemic to Antarctica.
Supplementary notes:
 The only penguin species that breeds during
the Antarctic winter, emperor penguins Pneumoconiosis Fund
trek 50–120 km (31–75 mi) over the ice to
 Rajasthan has announced the creation
breeding colonies which can contain up to
of a Pneumoconiosis Fund, which will be
several thousand individuals.
majorly financed by money from the District

RE
 It is listed as ‘near threatened’ in the Red Mineral Foundation (DMF). This Fund will
List of International Union for Conservation be used to finance a comprehensive policy
of Nature (IUCN). on the disease.

Recent demands:  The aim of the policy is to streamline the


strategy to deal with pneumoconiosis.
 Experts have demanded that the IUCN
status of species should be changed to  The Fund will be operating under
Social Justice and Empowerment


‘vulnerable’ from ‘near threatened’.
O
The experts also advocated that the emperor
penguin should be listed by the Antarctic
Treaty as a Specially Protected Species.

Department.
It will include pension for patients and
their families after the patient’s death and
their inclusion in the state’s social security
SC
 The 1959 treaty sets aside Antarctica as a schemes, which will be over and above the
scientific preserve, establishes freedom of compensation paid.
scientific investigation, and bans military
activity on the continent.  Rajasthan is one of the leading mining
states of India, with a distinction of having
more than 33,000 mine leases, the highest
96. Correct Option: (a) in the country. Most of these are sandstone
mines and quarries.
Explanation:
GS

Statement 1 is incorrect: It would be



setup in Bhopal.
98. Correct Option: (c)
Explanation:
Supplementary notes:
 Both statements are correct
e-Waste Clinic
Supplementary notes:
 The Bhopal Municipal Corporation (BMC)
and the Central Pollution Control Board C40
(CPCB) have signed a MOU to set up the
country’s first e-waste clinic in Bhopal.  C40 is a network of the world’s
megacities committed to addressing
 The clinic is being conceived in compliance climate change. C40 supports cities to
with the Solid Waste Management Rules, collaborate effectively, share knowledge
2016. and drive meaningful, measurable and
sustainable action on climate change.
 This would enable segregation, processing
and disposal of waste from both household  Around the world, C40 Cities connects 94
and commercial units. of the world’s greatest cities to take bold
climate action, leading the way towards a
 Electronic waste will be collected door-to-
healthier and more sustainable future.
door or could be deposited directly at the
clinic in exchange for a fee. Door-to-door  C40 cities are taking actions that reduces
collection will happen in two ways. Either global greenhouse gas emissions - together
separate carts for the collection of e-waste C40 member cities combined community
will be designed, or separate bins will be emissions represent 2.4 Gt of CO2.

41 PTS2020/PPP-16/112019/26
The chair  The survey was conducted by the AIIMS,
Delhi and Union Ministry of Health
 The Chair is the elected leader of the
organisation. and Family Welfare.

 The Chairmanship is a rotating position,  The survey was conducted between 2015
which has been held to date by the following and 2019.
C40 City Mayors:  The survey was conducted in 31 districts of
 Paris Mayor Anne Hidalgo (2016- 24 states and the results were extrapolated
present) for the entire country.
 Rio de Janeiro Mayor Eduardo Paes  India is the first country to launch
(2013-2016) National Program for Control of
Blindness in 1976. The aim of the program
 New York Mayor Michael R. Bloomberg
(2010-2013) is to reduce the blindness prevalence to
0.3% by 2020.
 Toronto Mayor David Miller (2008-
2010)  Key findings of the report

 London Mayor Ken Livingstone (2005-  Cataract is the leading cause of


2008) blindness in people above 50 years,
according to the survey.

RE
 The prevalence of blindness in India is
1.99%.
 Bijnor, Uttar Pradesh has the highest
population suffering from blindness.
 Around 3.67% of the districts are blind
and 21.82% suffer visual impairment.
O  Blindness is more pronounced
among illiterates as compared to
literates. Around 3.23% illiterates are
blind and 0.43% literates are blind.
SC
 Blindness is more prevalent in rural
population (2.14%) as compared to
urban (1.8%)
 Approximately 93% of cases of blindness
and 96.2% visual impairment cases in
this age group were avoidable.
Causes of blindness
GS

 Aphakia – Absence of lens in the eye


 Untreated non-infectious corneal opacity-
scarring of cornea
 Cataract related surgical complication was
the second highest cause for blindness.
 The outcomes of cataract surgery are not
good everywhere. Around 40% of cataracts
were done in government facilities. Rest of
the surgeries took place in private and non
– profit facilities. Of these, only 57.8% saw
99. Correct Option (a) good visual outcome. Cost was the biggest
barrier in accessing cataract surgeries.
Explanation:
 Around 22.1% of blindness was caused
 All statements are correct because of cost as a barrier. Around 22.1%
of blindness were caused due to lack of
Supplementary notes:
awareness
National Blindness and Visual Impairment
Survey, 2019 Barriers to accessing treatment

 Union Health Minister released the  Among men, the most important barriers
National Blindness and Visual Impairment are financial constraints (31%) and local
Survey 2019. reasons (21.5%).

PTS2020/PPP-16/112019/26 42
 Among women, local reasons (23.1%) and  ‘Eat Right Jacket’, and ‘Eat Right Jhola’
financial constraints (21.2%) were the most were launched to strengthen food safety
important barriers. administration and scale up the ‘Eat Right
India’ movement.

100. Correct Option (d)  Eat Right Jacket’ has a smart design to hold
tech devices like tablets/smart phone, a QR
Explanation: code and RFID tag for identification and
 All statements are correct tracking.
 Eat Right Jhola is a reusable cloth bag
Supplementary notes:
that is expected to replace plastic bags for
Safety Mitra Scheme shopping purpose in various retail chains.

™™™™™

RE
O
SC
GS

43 PTS2020/PPP-16/112019/26

Vous aimerez peut-être aussi